Добавил:
Upload Опубликованный материал нарушает ваши авторские права? Сообщите нам.
Вуз: Предмет: Файл:
Farmakologia_vse_testy.docx
Скачиваний:
19
Добавлен:
11.08.2019
Размер:
278.7 Кб
Скачать

Общая фармакология

1)Каким будет главный механизм всасывания лекарственного вещества, обладающего выраженными липофильными свойствами?

#.Пассивная диффузия

2)Выведение лекарственных веществ и их метаболитов через клеточную стенку происходит только путем:

#.Пассивной диффузии

3)Больному во время проведения оперативного вмешательства в качестве наркозного средства анестезиолог использовал азота закись, который имеет выраженные липофильные свойства. Какой механизм проникновения этого препарата через биологические мембраны?

#.Пассивная диффузия

4)Если химические вещества типа цианидов угнетают синтез АТФ, то какой из механизмов транспорта через клеточную мембрану при этом блокируется?

#.Активный транспорт

5)Оптимальный путь введения лекарств больному, который пребывает в коматозном состоянии:

#.Внутривенный

6)Как известно, энтеральный путь введения является наиболее распространенным и простым. Какой из названных способов введения лекарств не относится к энтеральным?

#.Вагинальный

7)Антиангинальный препарат нитроглицерин принимают сублингвально, он всасывается через слизистую полости рта, наступает быстрый эффект. Какой вид действия нитроглицерина проявляется при таком способе введения?

#.Резорбтивный

8)Известно, что вещество обладает выраженными липофильными свойствами. Какое из перечисленных фармакокинетических свойств будет характерно для него?

#.Наличие биотрансформации в печени

9)Машиной скорой помощи в больницу был доставлен мужчина, принявший большую дозу снотворого средства из группы барбитуратов (фенобарбитала). Состояние пострадавшего определено как барбитуровая кома. Среди комплекса мероприятий и лекарственных средств врач назначил введение натрия гидрокарбоната с целью ускорения выведения фенобарбитала через почки. Чем обосновано решение врача применить подщелачивание мочи в данной ситуации?

#.Увеличить полярность молекулы фенобарбитала

10)Молодая девушка 17 лет с целью суицида приняла большую дозу фенобарбитала. После прибытия на место врач скорой помощи быстро промыл желудок ввел бемегрид и раствор натрия гидрокарбоната внутривенно. С какой целью врач ввел натрия гидрокарбонат?

#.Для повышения почечной экскреции фенобарбитала

11)Что необходимо ввести больному при отравлении барбитуратами (имеют кислую реакцию) для ускорения их выведения.

#.NaHCO3

12)Больному с бактериальной инфекцией врач назначил сульфадимезин в таблетках, посоветовав запивать его щелочной минеральной водой. Чем обусловлена необходимость данной рекомендации?

#.Для профилактики кристаллизации ацетилированных производных препарата в почечных канальцах

13)При курсовом применении лекарственных средств, прочно связывающихся с белками крови и длительно циркулирующих в организме, возможна материальная кумуляция с развитием отравления организма. Укажите фармакокинетический критерий, который следует учитывать при назначении поддерживающих доз таких препаратов.

#.Константа элиминации

14)При лечении больных дигитоксином, что связывается с белками крови и продолжительное время циркулирует в организме, возможна материальная кумуляция с явлениями отравления. Какой фармакокинетический критерий прежде всего следует учитывать при назначении поддерживаемых доз препарата?

#.Коэффициент элиминации

15)Фармакологическое действие многих веществ развивается намного медленнее, чем их всасывание. Ниже перечислен ряд возможных явлений, определяющих скорость развития эффекта. Какое из них определяет скорость развития действия фенилина?

A.#.Продолжение действия белков-факторов свертывания, синтез которых нарушается

16)Какие лекарственные вещества легко проходят через тканевой барьер?

#.Те, которые хорошо растворяются в водной фракции плазмы

17)Известно, что вещество обладает выраженными гидрофильными свойствами. Какое из перечисленных фармакокинетических свойств будет характерно для него?

#.Быстрая почечная элиминация

18)У мужчины 36 лет с черепно-мозговой травмой дыхание слабое, пульс нитевидный, рефлексы отсутствуют. Какой путь введения пирацетама наиболее целесообразен в данном случае?

#.Внутривенный

19)Метаболизм лекарственных средств в эмбриональном периоде протекает значительно медленнее, чем в постэмбриональном. Указанная особенность фармакокинетики препаратов у плода обусловлена:

#.Функциональным несовершенством большинства ферментов или их отсутствием

20)Какой из указанных параметров является обязательным условием быстрого проникновения лекарственного вещества через гематоэнцефалический барьер?

#.Растворимость в липидах

21)К реакциям микросомного окисления относятся все названные, кроме:

#.Глюкуронирования

22)Как изменится фармакологическая активность препаратов с высоким сродством к белкам плазмы крови при возникновении гипоальбуминемии?

#.Повысится

23)Как известно, специфическая активность и токсичность лечебного средства обусловленная его свободной (не связанной с белками) фракцией. С какими белками сыворотки крови связывается большее количество лечебных средств?

#.Альбумин

24)При передозировке аспирина (ацетилсалициловой кислоты) применяют метод ощелачивания мочи. Этот метод направлен на то, чтобы:

#.Стимулировать образования ионизированной формы кислоты в просвете канальцев

25)Связывание лекарственного препарата с глюкуроновой кислотой во время его биотрансформации:

#.Повышает его растворимость в воде

26)Связывание антибиотика доксициклина с белками плазмы крови составляет 90%, за сутки из организма выводится 10 % препарата. Какой должна быть частота его назначения больному на протяжении суток?

#.1 раз в сутки

27)Что отображает такой фармакокинетический параметр лечебных средств как период полувыведения (Т1/2)?

#.Промежуток времени, за который концентрация препарата в плазме крови уменьшается на 50%

28)Во время назначения любого лекарственного препарата врач должен обратить внимание на его период полувыведения. Что можно определить на основе этого показания?

#.Кратность приема

29)Во время какой фазы фармакокинетического процесса лекарственные средства начинают действовать?

#.Распределение

30)Какой из путей выведения лекарственного вещества из организма является основным?

#.Почки

31)Показатель наиболее низкого уровня концентрации лекарственного средства в крови, способного создать терапевтический эффект имеет название:

#.Минимальная эффективная концентрация

32)Время, на протяжении которого, в организме присутствует эффективная концентрация лечебного средства имеет название:

#.Продолжительность действия

33)Биодоступность лекарственного препарата это:

#.Количество неизмененного вещества, которое содержится в плазме крови относительно исходной дозы препарата

34)Отметьте правильное утверждение. Какую дозу препарата следует назначать больным, возраст которых старше 60 лет?

#.Снижать на 30-50%

35)Больничная аптека перешла на другую марку дигоксинa (которую получила, как гуманитарную помощь), и в нескольких больных развилась инотоксикация дигоксином. Какую информацию нужно было предоставить врачам, чтобы предупредить это?

#.Про биодоступность каждой марки дигоксина

36)Больной Т. с болезнью Дюринга (буллезный гепертиформный дерматит) получает гидрокортизон и фенобарбитал. Каким образом введение фенобарбитала надо внести в режим назначения гидрокортизона?

#.Уменьшить дозу

37)Больной по поводу мегалобластической анемии, которая возникла после гастрэктомии, получил длительный курс лечения витамином В12. Препарат вводили внутримышечно. В чем преимущество парентерального пути введения витамина В12 от энтерального?

#.Эффективный при недостаточности гастромукопротеина

38)Больному с послеоперационной атонией кишечника ввели прозерин. Какой механизм лежит в основе действия данного препарата?

#.Антиферментный

39)Больному с эндогенной депрессией назначили амитриптилин. Какой механизм лежит в основе действия данного препарата?

#.Влияние на транспортные системы клеточных мембран

40)Какие свойства лекарств зависят от времени введения?

#.Все перечисленные свойства

41)Больной после перенесенного вирусного гепатита интересуется, существуют ли пути введения препарата в организм, минуя печень, с целью уменьшения функциональной нагрузки на нее. Какой путь введения и лекарственные формы можно предложить больному?

#.Ректальный

42)Мужчине 70 лет, страдающему хроническим бронхитом, назначен противокашлевой препарат - кодеин. Какой механизм обеспечивает противокашлевой эффект?

#.Центральный

43)Тестостерон и его аналоги увеличивают массу скелетных мышц, что позволяет использовать их для лечения дистрофий. Взаимодействием с каким клеточным субстратом обусловлено это действие?

#.Цитоплазматическими рецепторами

44)Терапевтический эффект некоторых психофармакологических средств развивается очень медленно (в течении 1-2 недель), что связывают с участием в его развитии изменения процессов транскрипции и биосинтеза постсинаптических мембранных рецепторов. Какому из названных веществ это присуще?

#.Трифтазину

45)Больному пневмонией назначены инъекции антибиотика. Укажите вид фармакотерапии в данной ситуации.

#.Этиотропная

46)Для уменьшения зубной боли больному рекомендовано принимать анальгин. К какому из перечисленных видов фармакотерапии относится этот метод лечения?

#.Симтоматическая

47)К какому виду терапевтического действия относят эффект гипотензивных средств?

#.Симптоматического

48)Больная 45 лет обратилась к врачу с жалобами на нарушение сна, что проявлялось ухудшением засыпания и пробуждения среди ночи. Врач назначил больной снотворное средство. Какой вид фармакотерапии использовал врач?

#.Симптоматическую

49)Пря тяжелом течении полиартрита больному с целью усиления эффекта были назначены диклофенак натрия и преднизолон, относящиеся соответственно к нестероидным и стероидным противовоспалительным средствам, которые уменьшают синтез провоспалительных простагландинов, угнетая различные ферменты. Какой вид взаимодействия препаратов здесь применен?

#.Непрямой синергизм

51)Больному, который отравился нитратом серебра, назначили для промывания желудка гипертонический раствор натрия хлорида. Какой механизи лежит в основе антидотного действия натрия хлорида?

#.Физико-химический

50) Во время оперативного вмешательства с применением миорелаксанта тубокурарина у пациента развилось нарушение дыхания, которое было ликвидировано после введения прозерина. Какому термину соответствует указанное взаимодействие между лекарствами?

#.Антагонизм

51) В стационар поступил больной с отравлением грибами, среди которых случайно оказался мухомор. Кроме промывания желудка, активированного угля и солеых слабительных внутрь, а также инфузионной дезинтоксикационной терапии, больному назначили инъекции атропина сульфата, в результате чего симптомы отравления значительно ослабились. Укажите тип взаимодействия мускарина (алкалоида мухомора) и атропина сульфата.

#.Прямой функциональный односторонний антагонизм

52)Какой вид синергизма наиболее эффективен при фармакотерапии?

#.Потенцирование

53) Больному была проведена операция по поводу ранения в живот с применением тубокурарина. В конце операции, когда дыхание восстановилось больному ввели гентамицин, неожиданно наступила остановка дыхания и длительное расслабление скелетных мышц. Какой эффект лежит в основе этого явления?

#.Потенцирование

54)Ребенок случайно выпил из флакона раствор, который его бабушка использовала для лечения глаукомы. Оказалось, что это был пилокарпина гидрохлорид. Врач назначил ребенку атропина сульфат. Какой механизм лежит в основе антидотного действия атропина?

#.Рецепторный

55)При отравлении грибами-мухоморами в качестве антидота используют атропин. Какой из перечисленных механизмов, принадлежащий атропину, приводит к устранению действия грибного яда - мускарина?

#.Рецепторный

56)Больному, который отравился серебра нитратом, назначили для промывания желудка гипертонический раствор натрия хлорида. Какой механизм лежит в основе антидотного эффекта натрия хлорида?

#.Физико-химический

57)Больной с язвенной болезнью желудка принимал антацидный препарат альмагель. Для лечения острого бронхита ему назначили антибиотик метациклин. Однако, на протяжении 5 дней температура не снизилась, кашель и характер мокроты не изменились. Врач пришел к выводу о несовместимости лекарств при их взаимодействии. О каком именно варианте несовместимости препарата идет речь?

#.Фармакокинетическая на стадии всасывания

58)Молодой женщине в состоянии тяжелого алкогольного опьянения врач скорой помощи среди прочих средств ввел внутримышечно раствор кофеина. Объясните на основании какого принципа взаимодействия данная манипуляция является целесообразной.

#.Физиологический антагонизм

59)Больному для кратковременного хирургического вмешательства произвели нейролептанальгезию путем введения фентанила и дроперидола. Что из перечиленных явлений обеспечивает достижение обезболивания, дастаточного для проведения операции?

#.Потенцирование

60)Больной лечившийся по поводу невроза сибазоном, почувствовал зубную боль. Врач назначил ему обезболивающее средство в дозе, меньшей от средней терапевтической. Какое явление принял во внимание врач, уменьшая дозу препарата?

#.Потенцирование

61)Больному язвенной болезнью назначили одновременный прием антацидного, спазмолитического и противовоспалительного средств. Выраженность симптоматики уменьшилась, боль в области желудка стала умеренной, но заживление язвы долго не наступало. Почему при взаимодействии назначенных средств не проявилось противовоспалительное действие?

#.Фармакокинетическая несовместимость

62)Больная без назначения врача принимает большое количество лекарственных средств. Как называется это явление ?

#.Полипрагмазия

63)Больной 38 лет, страдающий облитерирущим эндоартериитом сосудов нижних конечностей, получает фенилин в суточной дозе 60 мг/кг. В связи с проявлениями судорожного синдрома (в анамнезе ЧМТ) назначен фенобарбитал, после отмены которого у больного возникло носовое кровотечение. Данное осложнение связано с:

#.Торможением фенобарбиталом микросомального окисления в печени

64)Больному, принимавшему длительное время фенобарбитал, врач назначил диклофенак-натрия. Однако противовоспалительный эффект диклофинака оказался слабее от ожидаемого в результате фармакокинетического взаимодействия этих препаратов. Результатом каких процессов могло быть это взаимодействие препаратов?

#.Увеличение скорости элиминации и метаболизма

65)Унитиол является антидотом и используется при отравлениях солями тяжелых металлов. Как называется такой тип взаимодействия лекарственных веществ?

#.Химический антагонизм

66)Фармакологическая несовместимость лекарств, проявляющаяся на уровне специфического эффекта, называется:

#.Фармакодинамической

67)Развитие тяжелых сердечных аритмий при применении адреналина гидрохлорида для повышения артериального давления во время фторотанового наркоза служит примером осложнения, которое возникает на почве:

#.Фармакодинамической несовместимости

68)Уменьшение всасывания препаратов группы тетрациклина при их одновременном применении с антацидными средствами служит примером:

#.Фармакокинетической несовместимости

69)Больному, страдающему острым ринитом, врач назначил капли в нос 5% раствора эфедрина гидрохлорида по 2-3 капли (т.е. средние терапевтические дозы). После первого закапывания носовое дыхание у больного заметно облегчилось. Он стал закапывать в нос раствор эфедрина гидрохлорида каждые 20-30 минут. В результате было замечено резкое ослабление эффекта препарата. Врач констатировал явление тахифилаксии, развившееся на частое введение раствора эфедрина. Какой механизм лежит в основе развития явления тахифилаксии?

#.Истощение субстрата через который действует эфедрин

70)Больному с выраженной артериальной гипотензией (АД= 80/40 мм рт.ст.) ввели п/к 1 мл 5% раствора эфедрина гидрохлорида. Через 15-20 мин. в связи с незначительным изменением АД (повысилось до 90/40 мм рт. ст.) инъекцию эфедрина повторили, но АД не изменилось. Укажите, как называется ослабление фармакологического эффекта лекарственного средства при быстром повторном введении.

#.Тахифилаксия

71)Женщина 70 лет, страдающая бессонницей, длительное время принимает фенобарбитал. Какое наиболее вероятное осложнение лекарственной терапии может возникнуть при этом?

#.Толерантность

72)Больному с нарушением сна врач назначил препарат из группы барбитуратов. Через некоторое время больной почувствовал, что снотворный эффект от применения препарата уменьшился. Как называется это явление?

#.Толерантность

73)Мужчине 56 лет, который страдает бессонницей, был назначен фенобарбитал. Ритм сна нормализовался. Но постепенно на протяжении 3-4 недель действие препарата начало снижаться и бессонница возобновилась. Как называется явление, которое привело к снижению эффективности снотворного?

#.Толерантность

74)Тахифилаксия И это:

#.Ослабление эффекта при быстром повторном введении препарата

75)Какое явление характерно для кумуляции?

#.Усиление фармакологической реакции

76)Больному по поводу хронической сердечной недостаточности, назначили дигитоксин. Однако, через неделю после начала приема препарата у больного появились признаки интоксикации препаратом (брадикардия, тошнота, экстрасистолия), хотя его доза не превышала терапевтическую. Какое явление лежит в основе развития интоксикации?

#.Материальная кумуляция

77)У наркоманов после окончания действия наркотика развиваются тяжелые психоневрологические и соматические расстройства. Как называется этот симптомокомплекс?

#.Абстинентный синдром

78)Больному был проведен курс паллиативной терапии морфина гидрохлоридом на протяжении недели, после чего он стал требовать и в дальнейшем введения препарата. О каком явлении в данном случае идет речь?

#.Пристрастие

79)Больная 50 лет страдает бессонницей. В течение 3 месяцев она принимала разные снотворные средства: этаминал натрия, фенобарбитал, барбамил. После отмены препаратов больная стала раздражительной восстановилась бессонница, появилась агрессивность, потеря аппетита, тремор конечностей. Как называют ослажнение, возникшее у больной?

#.Перекрестная физическая и психическая зависимость

80)У больного с бронхиальной астмой и хронической сердечной недостаточностью с явлениями нарушения функции печени после курса лечения дигитоксином возникли брадикардия, нарушение сердечного ритма, симптомы ретробульбарного неврита. Какова причина указанных явлений?

#.Материальная кумуляция

81)Больной 60 лет после перенесенной операции находится на постельном режиме. После длительной терапии отваром коры крушины слабительный эффект препарата уменьшился. Какое явление обусловило этот процесс?

#.Привыкание

82)Больной продолжительное время лечился от хронического запора сенадексином. Однако, через несколько недель слабительный эффект препарата снизился. С чем это может быть связано?

#.Привыкание

83)Пациентка 57 лет, по специальности бухгалтер, в связи с хроническими запорами принимала по назначению врача экстракт крушины сухой, который на протяжении 2 недель оказывал позитивное лечебное действие, но позже его эффективность существенно снизилась. Определите, какая реакция развилась у больной в ответ на повторное введение препарата?

#.Толерантность

84)При повторном применении некоторых лекарственных средств к ним развивается медикаментозная зависимость. Из ниже перечисленных препаратов все имеют такое свойство, за исключением:

#.Нейролептики

85)Больному перед экстракцией зуба была проведена проводниковая анестезия новокаином (ранее препарат переносился хорошо), после введения которого появились отек и гиперемия вокруг места укола, зуд кожи, общая слабость, гипотензия, двигательное возбуждение. Определите, как называется возникшее осложнение?

#.Аллергическая реакция

86)Беременная женщина волновалась перед родами и на протяжении месяца принимала транквилизатор. У новорожденного наблюдается ослабление сосательного рефлекса. Какое действие реализуется при этом?

#.Фетотоксическое

87)Мужчине 18 лет по поводу флегмоны плеча была сделана внутримышечная инъекция пенициллина. После этого у него появились тахикардия, нитевидный пульс, АД снизилось до 80/60 мм рт.ст. Какой вид фармакологической реакции развился?

#.Анафилаксия

88)Беременная женщина волновалась перед родами и в течение недели принимала транквилизатор. У новорожденного наблюдалось ослабление сосательного эффекта. Какое действие реализуется при этом?

#.Фетотоксическое

89)Женщина во второй половине беременности принимала транквилизаторы бензодиазепинового ряда. Роды наступили в срок, протекали нормально, но ребенок родился с аномалиями развития ("заячья губа", полидактилия). Как назывется описанное действие препарата?

#.Тератогенное

90)Во время первого триместра беременности женщина с целью самолечения принимала большие дозы ретинола, что привело к развитию гипервитаминоза А. Далее беременность протекала нормально, но родившийся ребенок имел анатомические аномалии развития. Как называется это негативное действие лекарств?

#.Тератогенное

91)Беременная женщина (6 недель) принимала противодиабетический препарат бутамид для лечения сахарного диабета. Какой токсический эффект стал результатом неправильного применения лекарственного препарата во время беременности?

#.Тератогенный

92)При назначении беременным женщинам лекарственных средств необходимо предусмотреть развитие следующего побочного вида действия:

#.Всех перечисленных

93)У больного, длительно принимавшего глюкокортикоиды, в результате отмены препарата возникло обострение имеющегося заболевания, понижение артериального давления, слабость. Чем можно обьяснить эти явления?

#.Возникновением недостаточности надпочечников

94)На фоне приема алкоголя у больного, получавшего клофелин для лечения гипертонической болезни, появились явления резкого угнетения центральной нервной системы. С чем это может быть связано?

#.Потенцирование эффектов

95)Влияние определенных неблагоприятных факторов, в частности некоторых лечебных средств, которые предшествуют беременности, увеличивают риск рождения ребенка с генетическими недостатками. Как называется это действие?

#.Мутагенный эффект

96)Известно, что у лиц с генетически обусловленной недостаточностью глюкозо-6-фосфатдегидрогеназы эритроцитов в ответ на назначение некоторых противомалярийных препаратов может развиваться гемолиз эритроцитов. Как называется это проявление атипичной реакции на лечебные средства?

#.Идиосинкразия

2. Средства влияющие на перефирическую иннервацию.

97)Укажите механизм действия местноанестезирующих средств.

#.Блокируют натриевые каналы

98)Механизмы проводимости принципиально одинаковы во всех видах нервных волокон, но местные анестетики обладают преимущественным действием на чувствительные нервные волокна. От какого из названных факторов зависит это свойство анестетиков?

#.Диаметр волокон

99)Вещества с какими свойствами называются местноанестезирующими:

#.Устраняющие все виды чувствительности в результате паралича нервных окончаний или проводников в связи с понижением проницаемости мембран для ионов Na и K

100)При действии местных анестетиков на нервное волокно:

#.Понижается проницаемость мембраны для ионов натрия и калия, что приводит к невозможности образования потенциала действия

101)В результате влияния на какие элементы кожи и слизистых оболочек развивается терминальная анестезия?

#.Чувствительные нервные окончания

102)Какой из методов наиболее целесообразно использовать для местного обезболивания мягких тканей челюстно-лицевого участка у спортсмена 19 лет во время соревнований?

#.Местное охлаждение

103)У пациента 16 лет возникла необходимость провести операцию на мягком небе. Какой из методов обезболивания целесообразно избрать для этого?

#.Инфильтрационная анестезия

104)Почему новокаин не используют для терминальной анестезии?

#.Плохо проникает через неповрежденную кожу и слизистые

105)В очаге воспаления понижается сила местноанестезирующего действия новокаина. В условиях какого состояния в очаге воспаления возникает нарушение гидролиза соли новокаина и освобождение активного антисептика-основания?

#.Локального тканевого ацидоза

106)При использовании местных анестетиков в ротовой полости наблюдается сохранение тактильной чувствительности и исчезновение болевой. Какой из нижеперечисленных механизмов объясняет это явление?

#.Повышение болевого порога, снижение порога к механическим стимулам

107)При введении больному с целью проведения проводниковой анестезии средства, которое используется в хирургической стоматологии, возникли симптомы отравления, возбуждение ЦНС с последующим параличом, острая сердечно-сосудистая недостаточность (коллапс), в патогенезе которого имеет значение сенсибилизация к данному препарату. Возникли также аллергические реакции (зуд, отеки, эритема). Определите препарат.

#.Новокаин

108)Больному необходимо провести паранефральную блокаду по А.В. Вишневскому. Раствор новокаина какого процентного содержания необходимо ввести?

#.0,125 И 0,5%

109)Больному с тяжелой патологией печени врач решил назначить местный анестетик. Какой из ниже перечисленных местных анестетиков метаболизируется холинэстеразой крови?

#.Новокаин

110)Какие препараты из группы местных анестетиков нежелательно назначать одновременно с сульфаниламидами.

#.Новокаин

111)У больного выявлена генетически обусловленная недостаточность фермента бутирилхолинестеразы. Применение какого местного анестетика противопоказано больному?

#.Новокаина

112)При проведении врачом стоматологом инфильтрационной анестезии новокаином, у больного развился анафилактический шок. Какие мероприятия первой помощи необходимо провести врачу, с целью устранения шокового состояния?

#.Внутривенно ввести глюкокортикоиды, антигистаминные препараты и адреналина гидрохлорид

113)Какое средство, относящееся к местным анестетикам, следует использовать для купирования пароксизмальной желудочковой тахикардии при инфаркте миокарда?

#.Лидокаин

114)Укажите средство, применяемое для всех видов местной анестезии.

#.Лидокаин

115)Больному при удалении инородного тела из глаза необходимо произвести местную анестезию лидокаином. Каков механизм действия этого препарата?

a. #.Нарушает проницаемость мемебран клеток для Na+

116)В хирургическое отделение поступил больной по поводу острого аппендицита. Ему была назначена аппендектомия под местной анестезией. Однако из анамнеза выяснилось, что больной часто страдает аллергическими реакциями на лекарства. Выберите из указанных препаратов наиболее предпочтительный для инфильтрационной анестезии в данной ситуации.

#.Лидокаин

117)В офтальмологическое отделение обратился больной с травмой глаза, который имеет повышенную чувствительность к сульфаниламидным средствам. Какой из приведенных анестетиков следует применить для обезболивания конъюнктивы?

#.Лидокаин

118)Больному с повышенной чувствительностью к сульфаниламидным препаратам необходимо провести экстракцию зуба. Какой анестетик требуется использовать для проводниковой анестезии?

#.Лидокаин

119)Больной, который находится на больничном листе по поводу обострения гипертонической болезни, обратился к стоматологу с жалобами на боль в зубе, ассиметрию лица. Какое средство обезболивания в комплексе с другим препаратом необходимо предложить при удалении зуба?

#.Лидокаин с линкомицином

120)Больному для экстракции зуба под местной анестезией должны ввести лекарственное средство. Какое из перечисленных средств необходимо выбрать?

#.Ксикаин

121)Препарат мало растворим в воде, поэтому его применяют только для поверхностной анестезии в мазях, пастах, присыпках, ректальных суппозиториях, а также назначают внутрь в порошках при болях в желудке, рвоте. Определите препарат.

#.Анестезин

122)Врач во время стоматологической манипуляции использовал анестетик в виде порошка. Какое из перечисленных средств плохо растворяется в воде и используется в виде порошка?

#.Анестезин

123)Комбайнер во время сбора урожая ощутил резкую боль в глазу. Его доставили в больницу. Какое местноанестезирующее средство можно применить для удаления инородного тела?

#.Дикаин

124)Необходимо продлить действие местноанестезирующего вещества. Какой из предлагаемых адреномиметиков может быть применен для этого?

#.Мезатон

125)В хирургическое отделение стоматологической поликлиники поступил больной для экстракции зуба. Какой препарат необходимо добавить в раствор местного анестетика для пролонгирования его действия?

#.Адреналина гидрохлорид

126)Какое из отмеченных лекарственных средств необходимо добавить к раствору лидокаина для увеличения длительности его действия?

#.Адреналин

127)У пациента во время проведения проводниковой анестезии лидокаином развился анафилактический шок. Какой из перечисленных препаратов является препаратом выбора?

#.Адреналина гидрохлорид

128)Укажите механизм противовоспалительного действия вяжущих средств.

#.Образуют альбуминатную пленку, уменьшающую раздражение рецепторов

129)При нанесении на слизистые оболочки препараты вызывают осаждение белков слизи и уплотняют поверхностный слой слизистой оболочки, оказывая противовоспалительное дейсвие. Определите группу препаратов.

#.Вяжущие

130)Какой из препаратов солей тяжелых металлов обладает наиболее выраженным вяжущим действием?

#.Квасцы

131)Укажите принцип действия обволакивающих средств.

#.Образование защитного слоя на слизистых

132)Укажите основное показание к применению адсорбентов.

#.Интоксикация

133)В результате Чернобыльской аварии произошло загрязнение окружающей среды радионуклидами, накопление их в организмах людей. Укажите группу препаратов, способствующих выведению радионуклидов из организма.

#.Энтеросорбенты

134)Врач назначил пациенту мазь "Эфкамон", которая по механизму действия принадлежит к раздражающим веществам отвлекающего действия. Когда эта мазь противопоказана?

#.Повреждение кожи

135)Студент 3 курса при прохождении медицинской практики должен был поставить горчичники на спину больного. Для этого он смочил горчичники водой с температурой больше 600 С и поставил их на спину больному. Через полчаса студент обнаружил, что кожа больного на месте приложения горчичников не покраснела. С чем связано отсутствие эффекта горчичников?

#.Инактивацией мирозина

136)Антихолинэстеразные средства вызывают:

#.Спазм аккомодации

137)Мужчине 65 лет для лечения глаукомы назначили прозерин. Инактивацией какого соединения обьясняется способность прозерина снижать внутриглазное давление?

#.Ацетилхолинэстеразы

138)При проведении анестезии врач-анестезиолог передозировал дозу тубокурарина хлорида. Больному назначили прозерин. На чем основано действие этого препарата?

#.Уменьшение концентрации холинестеразы

139)У ребенка 5 лет врач обнаружил следующие симптомы: сильное двигательное возбуждение, галлюцинации, хриплый голос, расширенные зрачки, которые не реагируют на свет, сухость во рту, повышенную температуру, гиперемированную кожу, тахикардию, учащенное дыхание. Указанные симптомы появились после употребления ребенком плодов красавки. Какой препарат из представленных фармакологических групп лекарств необходимо назначить?

#.Антихолинэстеразные средства

140)В отделение интенсивной терапии доставлен больной с резкими болями в животе, рвотой, затруднением дыхания, одышкой. При обследовании больного кожа влажная, гиперсаливация, миоз, брадикардия, свистящее дыхание, мышечные подергивания. Из анамнеза установлено, что больной токсикоман, пользовался баллоном для уничтожения насекомых. К какой группе относится вещество, вызвавшее отравление?

#.Антихолинэстеразные

141)В приемное отделение доставлен больной со следующей симптоматикой: миоз, гиперсаливация, потливость, спазм бронхов, рвота и диаррея. Установлен диагноз: отравление фосфоорганическими соединениями. Какие препараты целесообразно включить в комплексную терапию?

#.Атропина сульфат и дипироксим

142)Больной 71 года доставлен в больницу скорой помощи с явлениями паралитической непроходимости. Какой из перечисленных препаратов следует назначить с целью стимуляции перистальтики?

#.Прозерин

143)У больного М., 59 лет, после нефрэктомии развился парез кишечника. Какое холинергическое средство из группы антихолинэстеразных средств следует ему назначить?

#.Прозерин

144)При атонии кишечника применяют:

#.Прозерин

145)В приемное отделение доставлен больной с жалобами на сухость во рту, светобоязнь и нарушение зрения. При осмотре кожа сухая, гиперемированная, зрачки расширены, тахикардия. Поставлен диагноз: отравление алкалоидами красавки. Какое из лекарственных средств целесообразно назначить?

#.Прозерин

146)Мальчик 5 лет перенес полиомиелит, в связи с чем у него наблюдается нарушение координации движений и мышечная слабость, преимущественно в правой ноге. Какое лекарственное средство следует ему назначить для улучшения нервно-мышечной передачи?

#.Прозерин

147)В эксперименте были получены гетерозиготные мыши с мутацией гена, кодирующего белок концевой пластинки скелетных мышц, который фиксирует ацетилхолинэстеразу в синапсе. Как изменится влияние прозерина на нервно-мышечную передачу у этих мышей?

#.Уменьшится

148)Женщина 63 лет получала инъекции галантамина для восстановления функций ЦНС после ишемического инсульта мозга. Состояние больной значительно улучшилось. Какой механизм действия этого средства?

#.Блокада ацетилхолинэстеразы

149)Пожилой мужчина перенес правосторонний ишемический инсульт. Какой препарат наиболее целесообразно применить для ускорения реабилитации и восстановления функции холинергических систем мозга?

#.Галантамин

150)Больной после инсульта парализован. Какой препарат нужно включить в его лечение, чтобы ускорить возобновление двигательных функций парализованных конечностей?

#.Галантамин

153)Для лечения гемипаралича, который связан с инсультом врач назначил препарат (третичный амин) из группы антихолинэстеразных средств. Назовите этот препарат.

#.Галантамина гидробромид

154)К врачу обратился мужчина с жалобами на зуд, гиперемию и отек глаз. Из анамнеза известно, что пациент лечится по поводу глаукомы. Какой из препаратов может вызвать такой эффект?

#.Галантамина гидробромид

153)С целью уменьшения саливации перед стоматологическим вмешательством врач дал больному для приема внутрь 10 капель 0,1% раствора атропина сульфата. Через 30 минут больной стал жаловаться на резкую болезненность в глазных яблоках, "туман" перед глазами, головную боль, сердцебиение. Устранить указанные симптомы удалось, использовав:

#.Физостигмин

154)При парентеральном введении крысе вещество вызывает возбуждение вегетативных ганглиев и усиливает секрецию адреналина хромаффинной тканью в кровь. Предварительная перерезка чревного нерва устраняет последний эффект вещества. Найдите его в приведенном перечне.

#.Пиридостигмин

155)В приемное отделение больницы доставлен больной с жалобами на головокружение, снижение остроты зрения, тошноту, слюнотечение и спастические боли в животе. Поставлен диагноз: отравление фосфорорганическими соединениями. Укажите специфические антагонисты в данной ситуации.

#.Атропина сульфат и дипироксим

156)В приемное отделение был доставлен больной со следующей симптоматикой: миоз, гиперсаливация, потливость, спазм бронхов, рвота и диарея. Был установлен диагноз: отравление фосфорорганическими соединениями. Какие препараты целесообразно включить в комплексную терапию?

#.Атропина сульфат и дипироксим

157)В клинической практике нередко встречаются случаи отравления фосфорорганическими инсектицидами, ядохимикатами, другими антихолинэстеразными средствами. Для лечения указанных отравлений применяется препарат дипироксим. Укажите фармакологическую группу, к которой он относится.

#.Реактиваторы холинэстеразы

158)В приемное отделение доставлен больной с признаками отравления фосфорорганическими инсектицидами. Какой препарат необходимо использовать в качестве средства первой помощи?

#.Дипироксим

159)М-холиномиметики применяются при:

#.Послеоперационной атонии кишечника

160)Выбрать препарат для больного, страдающего глаукомой.

#.Пилокарпина гидрохлорид

161)Больной, страдающей глаукомой был назначен пилокарпина гидрохлорид. Какой из приведенных ниже эффектов имеет наибольшее практическое значение для препарата?

#.Снижает внутриглазное давление

162)К офтальмологу обратился мужчина с жалобами на боли в правом глазу, ухудшение зрения. Исследование показало, что внутриглазное давление в этом глазу значительно повышено (до 28 eд). Какой препарат следует назначить больному в виде глазных капель?

#.Пилокарпин

163)К врачу стоматологу обратился больной с жалобами на сухость в полости рта. При дальнейшем обследовании был установлен диагноз: ксеростомия. Какое из лекарственных средств целесообразно применить?

#.Пилокарпина гидрохлорид

164)Для лечения ксеростомии стоматолог назначил препарат, который одновременно вызвал сужение зрачка и ухудшение зрения пациента. Назовите препарат.

#.Ацеклидин

165)К стоматологу обратился мужчина с жалобами на сухость полости рта, которая возникла после удаления миндалин. Врач назначил ацеклидин. Какое из перечисленных явлений препарат не вызывает?

#.Мидриаз

166)У больного, проходящего курс лечения в стационаре по поводу бронхиальной астмы, сопутствующим заболеванием является глаукома. Укажите группу препаратов, противопоказанных для лечения у данного больного.

#.М-холинолитики

167)У больного, проходящего курс лечения в стационаре по поводу язвенной болезни желудка, сопутствующим заболеванием является глаукома. Укажите группу препаратов, противопоказанных для лечения у данного больного.

#.Холинолитики

168)Препараты из этой группы врачи-стоматологи используют при операциях для снижения секреции слюнных желез, предотвращения ларингоспазма, головокружения, брадикардии. Назовите группу препаратов.

#.М-холиноблокаторы

169)После употребления пищи с грибами мухомора у взрослого человека через 15 - 20 минут началось возбуждение, резко усилилось потоотделение, возникли явления острого гастроэнтерита, зрачки резко сузились. Вскоре возникли судороги, брадикардия, значительная общая слабость, сонливость, развился колапс. Вызванный врач немедленно ввел больному антидот из группы:

#.М-холинолитики

170)У больного на приеме у стоматолога возникла гиперсаливация. Препарат какой группы снижает данное явление?

#.Холиноблокаторы

171)М-холинолитики имеют весьма широкое применение в современной практической медицине. Они применяются во всех следующих случаях, кроме.

#.Атония кишечника

172)Для проведения оперативного вмешательства на челюстно-лицевой области с целью уменьшения саливации применяют холинергические препараты. Какой из ниженазванных препаратов вы будете применять?

#.Атропина сульфат

173)В больницу поступил ребенок 6 лет с резко выраженными симптомами двигательного и речевого возбуждения, сухостью во рту, затруднением глотания, хриплым голосом. Кожа сухая, горячая. Зрачки резко расширены, светобоязнь, тахикардия. Из анамнеза установлено, что ребенок съел какие-то ягоды темно-фиолетового цвета. Воздействие какого из токсических веществ является причиной отравления?

#.Атропин

174)Ребенок 8 лет отравился грибом мухомором. Какой из препаратов ему наиболее целесообразно назначить?

#.Атропин

175)В токсикологическое отделение поступил мужчина 40 лет, который отравился инсектицидом - хлорофосом. Какой препарат, блокирующий периферические М-холинорецепторы, наиболее эффективен для лечения отравления?

#.Атропин

176)В токсикологическое отделение поступил ребенок 4 лет, который отравился мухоморами. Какой М-холиноблокатор необходимо использовать для оказания неотложной помощи?

#.Атропин

177)В результате неосторожного обращения с фосфорорганическим инсектицидом у рабочего возник бронхоспазм. Какой из предлагаемых бронхолитиков показан в этой ситуации?

#.Атропин

178)Для исследования глазного дна необходимо расширить зрачок. Укажите вещество, которое для этого используется в клинике:

#.Атропин

156)После применения у пациента 24 лет М-холиноблокатора атропина по поводу спазма аккомодации глаза у него повысился порог вкусовой чувствительности. Объясните механизм.

#.Уменьшилось слюноотделение

180)Что общего между атропином и метацином?

#.Блокируют М-холинорецепторы

181)Больному язвенной болезнью желудка на фоне повышенной кислотности желудочного сока в стадии обострения, в анамнезе которого имеется глаукома, необходимо назначить М-холинолитическое средство. Какому из них отдать предпочтение с учетом сопутствующих заболеваний?

#.Метацин

182)Больному с миастенией ввели прозерин. Через некоторое время появилась тошнота, рвота, диарея, подергивание поперечно-полосатых мышц. Назначьте физиологический антагонист прозерина.

#.Метацин

183)У больного миастенией после назначения прозерина появилась тошнота, диарея, подергивание мышц языка и скелетных мышц. Чем можно купировать интоксикацию?

#.Метацином

184)Врач-стоматолог, после введения больному ацеклидина с целью стимуляции слюноотделения, обнаружил появление цианоза губ, побледнение лица, потливость, хриплое, свистящее дыхание, приподнятость плечей, расширение грудной клетки, ее напряженность. Больной стал жаловаться на чувство недостатка воздуха и слабость. В анамнезе у больного  бронхиальная астма. Какой из нижеперечисленных препаратов, на основании механизма действия, следует ввести больному?

#.Метацин

185)Парентеральное введение вещества животному вызвало мидриаз, понижение функции желез внешней секреции, тахикардию, расширение бронхов. Какое это вещество, если известно, что оно не проникает через гемато-энцефалический барьер?

#.Метацин

186)Больному для купирования кишечной колики назначен атропина сульфат. Какое из названных заболеваний может служить противопоказанием в этом случае?

#.Глаукома

187)После операции на десне у больного на фоне гипертонической болезни развилась брадиаритмия. Какое лечебное средство целесообразно назначить?

#.Платифилина гидротартат

188)В урологическое отделение поступил мужчина 48 лет с симптомами почечной колики. У какого из нижеперечисленных лекарственных средств для купирования приступа основным действием является расслабление гладкой мускулатуры?

#.Платифиллин

189)Мужчина 40 лет доставлен в больницу с приступом печеночной колики. Какое из приведенных ниже средств наиболее эффективно расслабляет гладкие мышцы?

#.Платифиллин

190)В практической медицине при морской и воздушной болезни используют препарат, оказывающий угнетающее влияние на центры, связанные с вестибулярным аппаратом. Определите препарат.

#.Скополамин

191)Посоветуйте пациенту, страдающему морской болезнью препарат из группы М-холиноблокаторов.

#.Скополамин

192)Какой из перечисленных холинэргических препаратов имеет седативное сйствие на ЦНС?

#.Скопаламин

193)Укажите препарат, применяемый для профилактики укачивания в транспорте, в состав которого входит скополамин.

#.Аэрон

194)Мужчина 35 лет госпитализирован в отделение реанимации с отравлением угарным газом. Укажите препарат, возбуждающий жизненно важные центры продолговатого мозга рефлекторно, стимулируя Н-холинорецепторы синокаротидной зоны, и поэтому применяемый при данном отравлении.

#.Лобелин

195)Больному с гипертонической болезнью назначили ганглиоблокаторы в качестве гипотензивных. Какие эффекты лежат в основе снижения АД?

#.Блокада симпатических ганглиев

196)Больному гипертоническим кризом был введен ганглиоблокатор - бензогексоний. Каких явлений следует опасаться после введения препарата?

#.Ортостатической гипотензии

197)Больному в связи со значительным повышением АД была сделана п/к инъекция гипотензивного препарата. Через некоторое время при попытке встать с кровати больной почувствовал головокружение, потемнение в глазах и потерял сознание. Какой из перечисленных гипотензивных препаратов может вызвать развитие ортостатического коллапса?

#.Бензогексоний

198)При отеке легких на фоне системной артериальной гипертензии для уменьшения давления крови в малом кругу кровообращения применяют:

#.Бензогексоний

199)Мужчина 53 лет был доставлен в приемное отделение в тяжелом состоянии с жалобами на головную боль, головокружение, тошноту. АД- 220/120 мм рт.ст. После введения 1 мл 2,5% раствора бензогексония внутримышечно состояние больного улучшилось. Какой механизм действия этого препарата?

#.Блокада Н-холинорецепторов вегетативных ганглиев

200)Больную 58 лет готовили к операции по поводу холецистэктомии. В комплексе средств предоперационной подготовки был введен бензогексоний. Какой фармакологический эффект вызывает этот препарат, если больному дан наркоз?

#.Функциональная блокада висцеральных рефлексов

201)Какой препарат нейротропного действия относится к ганглиоблокаторам и показан для купирования гипертонического криза:

#.Пентамин

202)Чтобы избежать чрезмерной кровопотери в связи с операцией на щитовидной железе, врач решил использовать метод управляемой гипотонии с помощью внутривенного капельного введения лекарственного препарата. Укажите этот препарат.

#.Гигроний

203)В хирургии при выполнении операций на сердце и сосудах применяют метод управляемой гипотензии. Какой препарат из группы ганглиоблокаторов используется?

#.Гигроний

204)Во время оперативного вмешательства анестезиолог для управляемой гипотонии применил холинотропное средство. Какой препарат наиболее целесообразно применить в данном случае?

#.Гигроний

205)Больному 50 лет с повышенным АД (220/110 рт ст.) был введен внутривенно капельно 0,1% р-р гигрония. Каков механизм действия этого препарата?

#.Блокада Н-холинорецепторов

206)У роженицы, страдающей гипертонической болезнью, отмечалось раннее отхождение околоплодных вод и слабость родовых схваток. Какой препарат следует назначить для усиления родовой деятельности?

#.Пахикарпина гидройодид

207)К врачу обратился больной за консультацией по поводу действия нового препарата. Врач ознакомился со структурой бисчетвертичного производного и сказал, что это миорелаксант. Какая характерная черта структуры миорелаксантов?

#.Расстояние между атомами азота больше 6 атомов углерода

208)У больного во время оперативного вмешательства появились признаки передозировки тубокурарином. Препарат какой группы наиболее целесообразно использовать для ослабления эффекта передозирования?

#.Антихолинэстеразные

209)При комбинированном наркозе у больного, которому произведена резекция желудка, в качестве миорелаксанта использовался тубокурарина хлорид. Какое средство в качестве антагониста следует ввести больному для восстановления его самостоятельного дыхания?

#.Прозерин

210)В приемный покой поступил мужчина 45 лет с вывихом правого плечевого сустава. Какое лекарственное средство наиболее целесообразно применить для расслабления скелетных мышц с целью облегчения ортопедической манипуляции вправления вывиха?

#.Дитилин

211)В челюстно-лицевое отделение доставили больного с переломом нижней челюсти. Было решено сопоставление костей провести хирургическим методом под наркозом. После внутривенного введения миорелаксанта наблюдались кратковременные фибрилярные сокращения мышц лица больного. Какой миорелаксант был применен?

#.Дитилин

212)Больному со сложным переломом нижней челюсти необходимо cопоставить обломки. Для этого с целью расслабления мышц в/в ввели миорелаксант, который вызвал нервно-мышечную блокаду. Для устранения блокады были использованы эфедрин и кокарбоксилаза. Какой миорелаксант был введен?

#.Дитилин

213)С целью расслабления скелетных мышц и сопоставления отломков при переломе бедренной кости, больному был введен миорелаксант. При этом наступила остановка дыхания. После введения свежей цитратной крови дыхание восстановилось. Какой миорелаксант применили больному?

#.Дитилин

214)Для облегчения вправления вывиха в плечевом суставе с целью расслабления скелетных мышц больному был введен миорелаксант дитилин. При этом наступило "апноэ". Больному ввели прозерин, однако дыхание не восстановилось. Что необходимо ввести больному?

#.Свежую цитратную кровь

215)У больного после кратковременного оперативного вмешательства, проведенного с применением дитилина, в течение 30 мин отмечается отсутствие дыхания и не восстанавливается исходный тонус мышц. Какую помощь необходимо оказать больному?

#.Переливание крови

216)Средняя продолжительность действия дитилина, являющегося периферическим миорелаксантом деполяризующего типа действия по химической структуре близким к ацетилхолину, составляет 5-10 мин. При наследственно обусловленной недостаточности фермента, участвующего в биотрансформации указанного препарата, длительность расслабления скелетных мышц может достигать нескольких часов. Укажите этот фермент.

#.Холинэстераза

217)Больному для расслабления скелетной мускулатуры перед операцией был введен дитилин, вызвавший длительную миорелаксацию на срок около 6 часов вместо 5-7 минут. Это может быть связано с генетической недостаточностью:

#.Холинэстеразы плазмы крови

218)У больного, которому во время хирургического вмешательства, ввели миорелексант дитилин, угнетение дыхания и миорелаксация продолжались более 2 часов. Укажите фермент, отсутствие которого в сыворотке крови, обусловливает это состояние.

#.Бутирилхолинэстераза

219)В травматологический пункт машиной скорой помощи доставлен пострадавший в ДТП человек с диагнозом: закрытый перелом средней трети бедра со смещением. С целью репозиции костных отломком больному введено 10 мл 2% раствора дитилина в/в, в результате развилось длительное апноэ и миорелаксация. Дефицитом какого фермента обусловлена указанная фармакогенетическая ферментопатия.

#.Псевдохолинэстеразы

220)У какого препарата значительно увеличивается длительность эффекта при генетической недостаточности у пациента ложной холинестеразы?

#.Дитилин

221)После внутримышечного введения анальгина у больного развилась острая аллергическая реакция: удушье, боли за грудиной, падение АД до 40 мм рт.ст. Затем пульс исчез, наступила остановка сердечных сокращений. Какое средство фармакологической помощи следует применить в первую очередь?

#.Адреналин

222)После введения анальгина у больного началась острая аллергическая реакция, удушье, боль за грудиной. Пульс исчез, прекратились сердечные сокращения. Какое средство фармакологической помощи следует применить в первую очередь?

#.Внутрисердечно ввести адреналин

223)У больного анафилактический шок, вызванный новокаином, использованным для проведения местной анестезией. Какое средство подавляет высвобождение гистамина тучными клетками и применяется для купирования анафилактического шока.

#.Адреналин

224)У больного анафилактический шок. От какого из перечисленных адреномиметиков можно ожидать наибольший терапевтический эффект?

#.Адреналин

225)После проведенной внутрикожной пробы на чувствительность к пенициллину у больного через несколько минут развилась реакция гиперчувствительности, которую врач определил как анафилактический шок: больной потерял сознание, наступило удушье и брадикардия. Врач назначил средство для устранения этих симптомов. Какой препарат назначил врач?

#.Адреналин

226)Больному (в анамнезе сахарный диабет) врач поставил диагноз гипогликемическая кома и назначил раствор глюкозы в/в. Состояние больного не улучшилось. Что необходимо ввести больному дополнительно?

#.Адреналин

227)На изолированное сердце лягушки влияли нейротропными веществами. Какое из названных препаратов вызовет увеличение частоты сердечных сокращений в этих условиях?

#.Адреналин

228)Для продления и усиления местноанестезирующей активности новокаина, а также для уменьшения вероятности кровотечения во время проведения удаления зуба, врач-стоматолог применил препарат из группы адреномиметиков. Назовите препарат.

#.Адреналина гидрохлорид

229)В эксперименте на животном адреномиметик в терапевтических дозах вызвал повышение системного артериального давления, мидриаз и брадикардию. Какое вещество было введено?

#.Норадреналин

230)У больного острая гипотензия, вызванная передозировкой резерпина. Какое из предлагаемых веществ пригодно для её купирования?

#.Норадреналин

231)Больному с острым ринитом был назначен 5 % раствор эфедрина гидрохлорида (по 2-3 капли в нос 3 раза в сутки). Тем не менее больной закапывал нос каждые 20-30 минут. В результате эффект препарата, обусловленный развитием тахифилаксии, резко ослабился. Какой механизм лежит в основе развития этого явления?

#.Истощение депо медиатора в пресинаптических окончаниях

232)Больной хроническим бронхитом длительное время применяет эфедрин. С чем связан механизм действия этого препарата?

#.Стимуляция высвобождения норадреналина в синаптическую щель

233)Укажите показание к применению эфедрина.

#.Артериальная гипотензия

234)Больной с обструктивным бронхитом длительное время без контроля врача принимает эфедрин. Какие побочные эффекты могут наблюдаться у пациента?

#.Возбуждение ЦНС

235)Какие средства следует использовать в первую очередь при гипотензии вследствие сосудистой недостаточности периферического происхождения:

#.-адреномиметики

236)Вещества с какими свойствами следует использовать при сосудистой недостаточности:

#.Усиливающие симпатические влияния и возбуждающие -адренорецепторы

237)Для коррекции артериального давления при коллаптоидном состоянии больному был введен мезатон. Каков механизм действия данного препарата?

#.Стимулирует α-адренорецепторы

238)Офтальмолог с диагностической целью (расширения зрачков для обзора глазного дна) использовал 1% раствор мезатона. Мидриаз, вызванный препаратом, обусловлен:

#.Повышением содержания ИТФ и ДАГ

239)Больному 30 лет, во время операции под фторотановым наркозом у него развилась угрожающая жизни артериальная гипотензия. Какое из перечисленных лекарств следует назначить больному?

#.Мезатон

240)У больного под фторотановым наркозом резко упало артериальное давление. Какой препарат следует применить для восстановления уровня артериального давления?

#.Мезатон

241)В приемный покой доставлен больной с тяжелым отравлением неизвестным веществом в состоянии острой сосудистой недостаточности. Какой из указанных препаратов необходимо использовать для неотложной помощи?

#.Мезатон

242)У больного коллаптоидное состояние из-за снижения тонуса перифирических сосудов. Какой препарат наиболее эффективный в этой ситуации?

#.Мезатон

243)У пациента во время посещения стоматолога возникла резкая гипотензия. Какой из препаратов, которые влияют на адренергическую иннервацию вы предложите использовать для нормализации артериального давления?

#.Мезатон

159)Больному 68 лет для экстракции зуба была проведена проводниковая местная анестезия новокаином. Через 5-7 минут появились признаки снижения АД. Что нужно немедленно ввести для нормализации давления?

#.Мезатон

160)Во время удаления зуба в больного открылось кровотечение. Врач-стоматолог наложил тампон с лечебным средством и кровотечение уменьшилось. Какой препарат использовал врач?

#.Мезатон

246)Какие средства следует использовать для устранения бронхоспазма?

#.β-адреномиметики

247)У пациента в стоматологическом кабинете развился приступ бронхиальной астмы, который удалось снять сальбутамолом. К какой группе лекарственных средст принадлежит этот препарат?

#.β2-адреномиметики

248)Женщине 30 лет, с угрозой выкидыша был введен сальбупарт (сальбутамол), который снизил сократительную активность миометрия. Каков механизм действия этого лекарственного средства?

#.Активация β2-адренорецепторов

249)Благодаря чему, в первую очередь, 2-агонисты (например сальметерол) вызовут расслабление гладких мышц?

#.Ускорению инактивации киназы легких цепей (КЛЛ) миозина

250)Укажите бронхолитик из группы β2-адреномиметиков.

#.Сальбутамол

251)У женщины 40 лет развился приступ бронхиальной астмы. Какой из перечисленных бронхолитиков, эффективный для оказания неотложной помощи, относится к группе β2-адреномиметиков?

#.Сальбутамол

252)Больной 40 лет. Страдает бронхиальной астмой в течение 10 лет. Сопутствующее заболевание - сердечные аритмии (тахикардия). Какой адреномиметик целесообразно назначить для устранения бронхоспазма с учетом сопутствующего заболевания?

#.Сальбутамол

253)У больного во время посещения стоматолога случился приступ бронхиальной астмы. Что необходимо назначить больному для снятия бронхоспазма?

#.Сальбутамол

254)Укажите бронхолитическое средство, механизм действия которого заключается в возбуждении 2-адренорецепторов бронхов

#.Тербуталин

255)Больной, принимавший по поводу приступов бронхиальной астмы ингаляции изадрина отметил, что препарат вызывает сердцебиение и головную боль. Каким препаратом из перечисленных лучше заменить изадрин?

#.Тербуталин

256)На приеме у стоматолога у пациента начался приступ бронхиальной астмы. Врач стоматолог оказал помощь и приступ закончился. Пациенту дышать стало легче, но появилась тахикардия, головная боль, потливость. Какой препарат может вызвать такую реакцию?

#.Изадрин

257)Больной бронхиальной астмой не сообщил врачу, что у него бывают приступы стенокардии. Врач назначил препарат, после приема которого приступы бронхиальной астмы стали редкими, однако участились приступы стенокардии. Укажите, какой препарат был назначен.

#.Изадрин

258)Больная бронхиальной астмой для лечения заболевания ежедневно 3 раза в день принимала таблетки под язык. Через некоторый время она стала жаловаться на тахикардию, боли в области сердца, головокружение, снижения АД, быструю утомляемость. Какой препарат принимала больная?

#.Изадрин

261)В клинику поступила больная с жалобами на неприятные ощущения в области сердца, приступы острой слабости, затемнение сознания. Обследование больной и показания электрокардиограммы выявили наличие атриовентрикулярной блокады II степени. Какая группа препаратов будет применена для лечения атриовентрикулярной блокады?

#.-адреномиметики (изадрин)

260)Препарат, который вводили группе экспериментальных животных, вызвал незначительное повышение систолического артериального давления, однако снижал диастолическое. Предыдущее введение пропранолола (анаприлина) предупреждало указанные колебания артериального давления, вызванные препаратом. Действие какого средства изучалось?

#.Изадрина

261)Больному острой сердечной недостаточностью, у которого возникшая рефрактерность к сердечным гликозидам введен добутамин. Какой механизм действия этого препарата?

#.Стимуляция -адренорецепторов

262)У гомозиготных мышей с дефектом гена, кодирующего структуру 1-адренорецептора, исследовали действие веществ на инотропную функцию сердца. Какое вещество утратит свою активность в этих условиях?

#.Добутамин

263)Больному необходимо уменьшить тонус артериальных сосудов. Для этого целесообразно назначить:

#.-адреноблокаторы

264)Основной способ купирования гипертонического криза - быстрое и существенное снижение сосудистого сопротивления. Инъекционные препараты какой группы адренонегативных средств пригодны для этого?

#.-адреноблокаторы

265)Больной в течение последних нескольких месяцев страдает от повышенного артериального давления, которое часто переходит в гипертонические кризы. В результате клинического обследования был поставлен диагноз феохромоцитомы (опухоли мозгового слоя надпочечников), при которой продуцируется большое количество адреналина, ведущее к значительному повышению артериального давления. Какая группа препаратов наиболее эффективно поможет больному в данной ситуации?

#.-адреноблокаторы

266)Больному, страдающему эндартериитом, был назначен фентоламин. Через некоторое время у больного развилась тахикардия. Какая причина возникновения побочного эффекта?

#.Блокада пресинаптических 1-адренорецепторов

267)У больного Н. выявлено стойкое повышение артериального давления. При обследовании больного был установлен диагноз феохромоцитома. Какой из препаратов будет эффективен при данной патологии?

#.Фентоламин

268)Больному в результате клинического и лабораторного обследования был поставлен диагноз феохромоцитома (опухоль мозгового вещества надпочечников). Какой препарат из группы адреноблокаторов целесообразно применить при лечении данной патологии?

#.Тропафен

269)К врачу обратился больной 60 лет, который жалуется на появляющиеся в ночное время боли в нижних конечностях. Больной много и длительно курит. После обследования поставлен диагноз: болезнь Рейно (расстройство периферического кровообращения). Было назначено лечение, принимая которое больной стал замечать, что при вставании с постели у него кружится голова, вплоть до обморока. Врач констатировал это состояние, как ортостатический коллапс, результат побочного действия лекарства. Какой препарат, применяемый для лечения болезни Рейно, мог вызвать такое побочное действие?

#.Дигидроэрготоксин

270)Врач наблюдает больного 40 лет с сердечно-сосудистыми заболеваниями: гипертония с гиперкинетическим типом кровообращения и высоким содержанием ренина; стенокардия напряжения; синусовая тахикардия. Какая из перечисленных групп препаратов будет применена врачом для лечения этого больного?

#.-адреноблокаторы

271)Абсолютным противопоказанием для β-адреноблокаторов является следующее:

#.Бронхиальная астма

272)Больной с артериальной гипертензией был назначен анаприлин, который быстро нормализовал АД. Какой механизм действия этого препарата?

#.Неизбирательно блокирует β1- и β2-адренорецепторы

273)Больному гипертонической болезнью с сопутствующим обструктивным бронхитом в составе комплексной терапии назначили анаприлин. Через некоторое время у него появились приступы бронхоспазма. С чем связано описанное явление?

#.Блокада β2 -адренорецепторов бронхов

274)Терапия анаприлином положительно повлияла на динамику болезни у женщины 44 лет, страдающей стенокардией. Какой главный механизм действия этого препарата?

#.Блокада β-адренорецепторов и снижение потребности миокарда в кислороде.

275)Терапия анаприлином оказала положительное влияние на динамику болезни у женщины 44 лет, страдающей стенокардией при наличии тахиаритмии. Какой принцип антиангинального действия этого препарата?

#.Уменьшение потребности миокарда в кислороде, вследствие уменьшения частоты и силы сердечных сокращений

276)Терапевтический эффект-блокаторов [пропранолол] во время стенокардии обусловлен следующим действием:

#.Снижение потребности миокарда в кислороде

277)При объективном обследовании у больного Н. 46 лет обнаружили повышение уровня АД до 145/95 мм рт.ст. при частоте сердечных сокращений 50 уд/мин. Назначение гипотензивных препаратов какой группы нецелесообразно и почему?

#.Бета-адреноблокаторов: за счет отрицательного хронотропного и дромотропного эффектов

278)Женщина 42 лет поступила в кардиологическое отделение с диагнозом стенокардия. Приступы возникают 3-4 раза в день. АД=170/100 мм рт.ст., пульс 84 уд./мин. На ЭКГ - экстрасистолы. Какой из препататов наиболее целесообразно назначить больной для лечения?

#.Анапpилин

279)Антигипертензивное средство повышает тонус миоцитов сосудов скелетных мышц и увеличивает их сопротивление току крови. Какое из названных веществ имеет такое свойство?

#.Анаприлин

280)Больной с ишемической болезнью сердца не сообщил врачу, что у него бывают приступы бронхоспазмов. Врач назначил препарат, после приема которого приступы стенокардии стали реже, а приступы бронхоспазмов стали чаще. Какой препарат был назначен?

#.Анаприлин

281)Больному для лечения ИБС был назначен бета-адреноблокатор, спустя некоторое время у него появился кашель, бронхоспазм. У какого из пересчитанных средств есть такое побочное действие?

#.Анаприлин

282)Больной с гипертонической болезнью имеет сопутствующее заболевание хронический бронхит с астматическим компонентом. Какое из перечисленных гипотензивных средств (в связи с его влиянием на бронхи) противопоказано пациенту?

#.Анаприлин

283)Больному с мерцательной аритмией, в анамнезе у которого бронхиальная астма, надо назначить противоаритмическое средство. Какой препарат из этой группы протипоказан больному?

#.Анаприлин

284)Больному гипертонической болезнью был назначен препарат для снижения АД из группы адренотропных средств. Через некоторое время у больного давление нормализовалось, но развилась брадикардия до 50 ударов в минуту и атривентрикулярная блокада II степени. Какой препарат был назначен?

#.Анаприлин

285)Больному 60 лет. Страдает гипертонической болезнью. Артериальное давление 180/90 мм рт.ст. Состоит на диспансерном учете по поводу сахарного диабета. Принимает сахароснижающие средства производные сульфанилмочевины. По совету знакомого для лечения гипертонии стал принимать препарат, который вызвал пролонгирование лекарственной гипогликемии. Какой из перечисленных препаратов, принимаемых больным в качестве самолечения, мог вызвать такое побочное действие?

#.Анаприлин

286)Больной состоит на диспансерном учете по поводу гипертонической болезни. Имеет сопутствующие заболевания: мерцательная аритмия, стенокардия, хронический бронхит. Больному необходимо назначить препарат из группы -адреноблокаторов. Какой препарат будет показан больному с учетом сопутствующих заболеваний?

#.Метопролол

287)Больному, страдащему ИБС, перед удалением зуба, врач порекомендовал провести профилактический курс лечения антиангинальным препаратом из группы адреноблокаторов. Какой из препаратов было рекомендовано больному?

#.Метопролол

288)Больному 50 лет, диагноз: ИБС, кардиосклероз, мерцательная аритмия. Укажите, какой из препаратов необходимо назначить больному?

#.Атенолол

289)У больного гипертоническая болезнь с брадиаритмией. Назначьте кардиоселективный бета-блокатор с внутренней симпатомиметической активностью:

#.Талинолол (Корданум)

290)Фармакологическое действие многих веществ развивается намного медленнее, чем их всасывание. Ниже перечислен ряд возможных явлений, определяющих скорость развития эффекта. Какое из них определяет скорость развития действия резерпина?

#.Опустошение депо медиатора

291)Больной, страдающий гипертонической болезнью, принимает комплексное лечение, включающее применение нейротропных, миотропных, диуретических препаратов. В последнее время стал жаловаться на боли в эпигастральной области, понос, изжогу. Врач связывает это с применением лекарственных средств. Какой препарат вызывает у больного перечисленные побочные эффекты?

#.Симпатолитики (октадин)

292)Больному с артериальной гипертензией (160/90 мм рт. ст.) назначено гипотензивное средство. Через час после приема внутрь первой таблетки он почувствовал некоторое усиление головной боли, измерение АД показало величину 170/100 мм рт. ст. Больной лег в постель, через 2 часа после очередного измерения АД его величина была 140/90 мм рт. ст. Укажите гипотензивное средство, первоначально вызывающее повышение АД, которое было назначено больному.

#.Октадин

293)Больной гипертонической болезнью был назначен препарат, механизм гипотензивного действия которого обусловлен уменьшением содержания норадреналина в гранулярных депо окончаний симпатических нервов. Какой препарат обладает таким механизмом действия?

#.Резерпин

294)Больной с гипертонической болезнью длительное время лечился лекарственными препаратами из группы алкалоидов раувольфии. Стал жаловаться на изжогу, боли в эпигастральной области, плохое настроение. Врач при обследовании поставил диагноз: язвенная болезнь желудка, депрессивное состояние. Какой лекарственный препарат принимал больной?

#.Резерпин

295)У женщины 70 лет, страдающей гипертонической болезнью, длительное применение лекарственного препарата привело к развитию депрессии. Укажите этот препарат

#.Резерпин

296)Больной длительное время лечится по поводу гипертонической болезни препаратами, содержащими резерпин. Последние 2-3 месяца его стали беспокоить боли в области желудка, изжога, тошнота. После обследования поставлен диагноз: гастрит с повышенной кислотообразующей функцией. Укажите группу препаратов, которые являются этиотропными средствами лечения гастрита в данной ситуации.

#.М-холиноблокаторы

297)В медицинской практике применяется значительное число средств для наркоза. Укажите препарат, при применении которого наблюдается длительная стадия возбуждения.

#.Эфир для наркоза

298)Для проведения оперативного вмешательства по поводу острого флегмонозного аппендицита врач применил средство для наркоза, которому свойственна длительная стадия возбуждения. Какой препарат был введен пациенту?

#.Эфир для наркоза

299)В стоматологии широко используют средства для наркоза как при хирургических вмешательствах, так и местно. Назовите наркозное средство, котрое применяется местно для обработки кариозной полости.

#.Эфир

55)Мужчине 40 лет во время операции на органах брюшной полости проводили ингаляционный наркоз с помощью летучей жидкости. Hаркоз наступил через 5 мин. от начала ингаляции, без стадии возбуждения. Во время наркоза отмечалось снижение АД, брадикардия и аритмия со стороны сердца. Пробуждение после наркоза наступило быстро, без послеоперационной депрессии. Какое средство для наркоза было использовано?

#.Фторотан

301)У больного, находящегося в состоянии наркоза, во время операции, врач обнаружил, что АД прогрессивно падает. Было назначено введение адреналина. Появилась фибрилляция желудочков. Укажите, какой препарат для наркоза может вызвать подобное осложнение.

#.Фторотан

302)Какое из средств для наркоза в процессе биотрансформации образует ионы фтора и может нарушать функцию почек?

#.Фторотан

303)После повторного оперативного вмешательства с использованием такого же общего анестетика у больного возникло острое поражение печени (гепатит). Какой общий анестетик наиболее вероятно мог вызвать данную патологию?

#.Фторотан

304)Во время фторотанового наркоза у больного возникла выраженная брадикардия с угрозой остановки сердца. Какой из перечисленных препаратов необходимо использовать для ускорения частоты сердечных сокращений в условиях наркоза, который должен продолжаться?

#.Атропин

305)Женщина 22 лет поступила в стационар для родоразрешения. Родовая деятельность удовлетворительная, но сопровождается сильными болями. Выберите препарат для обезболивания родов, не угнетающий родовую деятельность.

#.Азота закись

306)Больному с переломом нижней челюсти с целью обезболивания во время операции ввели раствор тиопентал-натрия. Каков принцип действия этого препарата?

#.Угнетение межнейронной передачи возбуждения в ЦНС

307)Для вводного наркоза медсестра должна ввести пациенту 55 лет, которому планируется провести резекцию желудка, внутривенно раствор тиопентала натрия. Укажите опаснейший нежелательный эффект тиопентала натрия, которого необходимо опасаться при его введении.

#.Остановка дыхания

308)Больному для вводного наркоза внутривенно ввели тиопентал натрия, после чего развились ларингоспазм и гиперсаливация. Какой препарат мог бы предупредить нежелательные эффекты?

#.Атропина сульфат

309)Определите средство для наркоза. Является синтетическим аналогом естественного метаболита, обнаруженного в ЦHС. Хорошо проникает через ГЭБ. Оказывает седативное, снотворное, наркотическое и антигипоксическое действие. Анальгетический эффект выражен в небольшой степени. Вызывает выраженную миорелаксацию. Повышает устойчивость тканей мозга и сердца к гипоксии.

#.Hатрия оксибутират

310)Определите препарат:обладает седативным, снотворным, наркозным, антигипоксическим, миорелаксирующим действием, используется как средство для наркоза и снотворное.

#.Натрия оксибутират

311)У больного травматический отек мозга, гипоксические судороги. Какое средство рационально использовать для купирования судорг?

#.Натрия оксибутират

312)В нейрохирургическое отделение поступил больной с травмой черепа. Прогрессивное нарастание неврологической симптоматики позволяет поставить диагноз ушиба мозга, являющегося причиной нарастающего отека, ведущего к сдавлению и гипоксии мозга. Назначьте больному лечебный наркоз, который можно использовать и как средство борьбы с гипоксией мозга.

#.Натрия оксибутират

313)В нейрохирургическое отделение доставлен больной. Из анамнеза: был избит, получил множественные ушибы головы и лица. Отмечается психомоторное возбуждение. Жалобы на резкую головную боль. Hа рентгенограммах не обнаружено нарушения целостности костей черепа. Установлен диагноз: закрытая черепно-мозговая травма. Выберите из данных препаратов наиболее предпочтительный в данной ситуации.

#.Натрия оксибутират

314)Ребенку 3-х лет назначена операция по поводу врожденного порока сердца. Какой из перечисленных препаратов с наркотическим и антигипоксическим действием необходимо использовать для вводного наркоза, учитывая, что у ребенка сильно затруднен доступ к венам?

#.Натрия оксибутират

315)В травмпункт доставлен больной по поводу свежего, неосложненного вывиха плечевого сустава. Выберите средство для наркоза быстрого, но кратковременного действия для амбулаторного вправления вывиха плечевого сустава.

#.Кетамин

318)Для обезболивания манипуляции, связанной с обработкой ожоговой поверхности ввели внутривенно препарат для кратковременного наркоза. Hаркоз наступил через минуту. Во время наркоза наблюдалось повышение АД (на 20-30%), тахикардия, повышение тонуса скелетных мышц, рефлексы сохранены. После выхода из наркоза у больного отмечались дезориентация, зрительные галюцинации. Какой препарат ввели больному?

#.Кетамин

319)Мальчик 15 лет с политравмой, бес сознания, АД 95/70 мм рт.ст., пульс 72 удара в минуту, дыхание поверхностное с частотой 30 за мин. Какое средство для наркоза следует избрать?

#.Кетамин

316)Экстракцию зуба у больного хирург-стоматолог выполнил под общей анестезией. Какое из перечисленных средств можно использовать в этом случае для кратковременного наркоза?

#.Кетамин

317)Для обезболивания биопсии больному внутривенно ввели препарат, который через минуту вызвал наркоз длительностью около 5 минут. Во время наркоза отмечались спонтанные движения, незначительное снижение АД, кратковременная остановка дыхания, которая быстро восстановилась. Какой препарат использовали?

#.Пропанидид

318)Пропанидид широко используется в амбулаторной стоматологической практике. Определите противопоказание для его использования.

#.Шок

319)Для премедикации используются лекарственные препараты, кроме:

#.Кофеин

320)При оперативном вмешательстве для общей анестезии использована комбинация ингаляционного анестетика и производного барбитуровой кислоты. Какая из перечисленных комбинаций была использована?

#.Тиопентал натрия + фторотан

321)С целью согревания после переохлаждения больной использовал раствор этилового спирта внутрь. Как влияет спирт этиловый на терморегуляцию?

#.Повышает теплоотдачу

322)Известно, что одна и таже доза алкоголя вызывает разные степени алкогольного опьянения у разных людей. Это связано с:

#.Генетически обусловленными особенностями синтеза алкогольдегидрогиназы.

323)Отметить концентрацию этилового спирта,обладающую наиболее активным противомикробным действием при наличии белка в среде:

#.70 %

324)В наркологическом отделении психоневрологической больницы больному назначен тетурам. Какое из перечисленных заболеваний может быть показанием к его применению?

#.Алкоголизм

325)Укажите механизм действия тетурама, применяемого для лечения хронического алкоголизма.

#.Нарушает окисление спирта этилового на стадии ацетальдегида

326)Для лечения больного хроническим алкоголизмом врач назначил тетурам в сочетании с небольшими дозами алкоголя. Каков механизм действия данного препарата?

#.Угнетение активности фермента ацетальдегидоксидазы

327)Укажите рвотное средство центрального действия для использования в комплексе лечения хронического алкоголизма.

#.Апоморфина гидрохлорид

328)Укажите характер изменений фазовой структуры сна под влиянием снотворных:

#.Укорочение фазы сна с быстрым движением глаз (быстроволнового)

329)Больной, принимавший по случаю бессоницы фенобарбитал, отметил снижение его действия. Это может быть связано с:

#.Повышением скорости синтеза микросомальных ферментов

330)В больницу доставили женщину 57 лет в коматозном состоянии. Из анамнеза известно, что она страдала бессонницей. При обследовании выявлено: угнетение дыхания, снижение АД, сердечная недостаточность, снижение температуры тела, угнетение сухожильных рефлексов. Какой препарат вызвал отравление?

#.Фенобарбитал

331)С целью облегчения процесса засыпания больной принял несколько таблеток фенобарбитала. Вскоре он потерял сознание, артериальное давление снизилось, дыхание резко угнетено. Какой специфический антагонист необходимо использовать?

#.Бемегрид

332)У больного в результате материальной кумуляции барбитуратов появились признаки интоксикации. Какие средства могут снизить концентрацию барбитуратов в ЦНС?

#.Hатрия гидрокарбонат

333)Что необходимо ввести больному при отравлении барбитуратами?

#.NaHCO3

334)Больной 68 лет обратился в поликлинику с жалобами на расстройство психики, которое он связывает с продолжительным употреблением фенобарбитала. Какой препарат ему следует назначить в качестве снотворного средства?

#.Нитразепам

335)Назовите снотворное средство не нарушающее структуру сна.

#.Нитразепам

336)Какой из следующих препаратов вызовет наименьшую супрессию парадоксального сна?

#.Нитразепам

337)После пережитого нервного потрясения женщина на протяжении нескольких дней плохо спит. Какому снотворному препарату нужно отдать предпочтение для лечения такой формы бессонницы?

#.Нитразепам

338)Больному при бессоннице, вызванной эмоциональными расстройствами, врач назначил средство, которое вызывает сон за счет транквилизирующего действия. Какой снотворный препарат был назначен пациенту?

#.Нитразепам

339)Больная 60 лет обратилась к врачу с жалобами на поверхностный кратковременный сон с частыми пробуждениями в связи с волнениями, переживаниями и появлением чувства тревоги. Был поставлен диагноз: старческая бессонница. Сделайте рациональный выбор снотворного средства в данной ситуации.

#.Нитразепам

340)Больной 18 лет обратился к врачу с жалобами на бессонницу, проявляющуюся в трудном засыпании, в результате чего он не высыпается и на другой день чувствует усталость, разбитость, трудно усваивает учебный материал. При обследовании установлено: больной раздражителен, эмоционально неустойчив, резкий дермографизм, пульс и АД колеблются во время беседы. Врач определил, что бессонница связана с неврозоподобым состоянием и вегетососудистой дистонией. Сделайте рациональный выбор снотворного средства.

#.Нитразепам

341)Больной эпилепсией длительное время принимал фенобарбитал в суточной дозе 0,4 г. В последнее время у него участились приступы, отмечается угнетенное настроение. Чем вызвано ухудшение состояния больного?

#.Индукция ферментов монооксигеназной системы печени

342)У больного после травмы периодически отмечаются генерализованные тонико-клонические судороги с потерей сознания, которые потом сменяются общим угнетением центральной нервной системы. Какой препарат следует назначить больному?

#.Фенобарбитал

343)Больному эпилепсий, у которого диагностированы большие судрожные припадки, назначили дифенин. Каков механизм действия данного препарата?

#.Блокада натриевых каналов

344)К врачу-стоматологу обратился больной с жалобами на разрастание и болезненность десен. На основании обследования полости рта врач поставил диагноз гиперпластического гингивита. Из анамнеза было установлено, что больной длительно принимал препарат, назначенный ему невропатологом по поводу эпилепсии. Какой препарат принимал больной?

#.Дифенин

345)Противосудорожные препараты имеют широкое применение при лечении невралгии тройничного нерва из-за выраженного обезболивающего действия. Какой это препарат?

#.Карбамазепин

346)У больного, длительное время страдающего эпилепсией с большими припадками, после перенесенного тяжелого приступа припадки приобрели затяжной характер и стали следовать один за другим, в связи с чем он был доставлен машиной скорой помощи в больницу, где был поставлен диагноз эпилептического статуса. Укажите, какое средство является препаратом выбора для купирования эпилептического статуса.

#.Диазепам

347)У больного во время удаления зуба произошел приступ судорог. Какой препарат целесообразно ему назначить?

#.Сибазон

348)Врач обсуждал с коллегами применение нового противоэпилептического препарата - натрия вальпроата. Каков механизм действия этого препарата?

#.Угнетение активности фермента ГАМК-трансферазы

349)Больной обратился к врачу с жалобами на ригидность мышц, скованность движений, постоянный тремор рук. На основании обследования врач поставил диагноз : болезнь Паркинсона. Из анамнеза установлено, что больной страдает также глаукомой. Сделайте рациональный выбор препарата.

#.Леводопа

350)Больного паркинсонизмом долгое время лечили препаратами с центральным холинолитическим механизмом действия, эффективность которых постепенно уменьшалась. На какой из перечисленных препаратов надо перейти для усиления антипаркинсонического эффекта?

#.Леводопа

351)Больному паркинсонизмом назначено лечение. Какой из препаратов будет эффективным?

#.Леводопа

352)Для лечения болезни Паркинсона врач назначил циклодол. Какой механизм противопаркинсонического действия препарата?

#.М-холиноблокирующие действие

353)Больная Н., 65 лет, с болезнью Паркинсона, эффективно лечится циклодолом. Механизм действия этого препарата?

#.Блокада центральных холинорецепторов

354)В неврологическое отделение поступил пациент, страдающий болезнью Паркинсона. Укажите препарат, угнетающий холинергические влияния, который применяется для лечения этого заболевания.

#.Циклодол

355)Больной, принимавший длительное время резерпин для лечения гипертонической болезни, стал отмечать у себя мышечную слабость, ограничение движений. Поставлен диагноз медикаментозного паркинсонизма. Какой препарат необходимо назначить больному для снижения этого побочного эффекта?

#.Циклодол

356)К стоматологу обратился больной со жалобой на зубную боль. Во время предыдущего разговора врач узнал, что больной страдает паркинсонизмом и потому принимает М-холинолитик циклодол. Какой нежелательный эффект циклодола у больного может ожидать врач-стоматолог?

#.Сухость во рту

357)В приемное отделение поступил больной в бессознательном состоянии: кожа холодная, зрачки сужены, дыхание угнетено, отмечается периодичность по типу Чейн-Стокса, АД понижено, мочевой пузырь переполнен. Чем произошло отравление?

#.Наркотическими анальгетиками

358)С целью аналгезии могут быть использованы вещества, имитирующие эффекты морфина, но вырабатывающиеся в ЦНС. Укажите такое вещество.

#.b-эндорфин

359)Больному с острыми болями в результате черепно-лицевой травмы после введения морфина стало значительно легче. Какой из перечисленных механизмов действия обеспечивает противошоковый эффект морфина?

#.Стимуляция опиатных рецепторов

364)Во время подготовки больного к операции резекции верхней челюсти врач однократно назначил раствор морфина гидрохлорида. Какой эффект можно наблюдать у больного во время однократного использования морфина?

#.Обстипация (запор)

361)Почему морфин нельзя использовать при переломах верхней челюсти?

#.Повышает внутричерепное давление

362)Какое из побочных эффектов морфина способствует его передозировке наркоманами?

#.Толерантность

363)При осмотре больного отмечено резкое сужение зрачков, сонливость, редкое дыхание по типу Чейна-Стокса, задержка мочи, замедление сердечного ритма. Какое вещество вызвало отравление?

#.Морфин

364)Для купирования болевого синдрома пациенту с инфарктом миокарда врач назначил аналгетический препарат. Состояние больного улучшилось, но со временем появилась эйфория, миоз. Врач отметил угнетение дыхания. Какое лекарственное средство было назначено?

#.Морфин

365)Больному, доставленному в отделение хирургической стоматологии после автокатастрофы ввели лекарственное средство растительного происхождения И алкалоид, производное пиперидинфенантрена, обладающий анальгезирующим действием. Вызывает сон, характеризующийся чуткостью и яркостью сновидений. Угнетает дыхательный и кашлевой центры. Вызывает лекарственную зависимость. В стоматологической практике используется при травмах и операциях на челюстно-лицевой области. Определите препарат.

#.Морфина гидрохлорид

366)У мужчины 84 лет диагностирован иноперабельный бронхогенный рак с метастазами в позвоночник. Какой препарат наиболее целесообразно использовать для уменьшения болевого синдрома у пациента?

#.Морфин

367)Больному с неоперабельным раком легкого, сопровождающимся трудно переносимыми болями, врач назначил болеутоляющее средство. На этом фоне у больного возникли явления непроходимости кишечника. Какой из болеутоляющих препаратов мог вызвать это осложнение?

#.Морфин

368)Женщине 25 лет с явлениями острой интоксикации морфином был введен налоксон, вызвавший быстрое улучшение ее состояния. Каков механизм действия этого препарата?

#.Блокада опиоидных рецепторов

369)При отравлениях лекарственными веществами и ядами широко прменяется антидотная терапия. В реанимационное отделение поступил больной с острым отравлением опием. Укажите специфический антагонист морфина, который следует назначить при отравлении опием.

#.Налоксон

370)У больного признаки острого отравления морфином И резкий миоз, потеря сознания, падение АД, дыхание Чейн-Стокса. Выберите антидот, показанный при данном отравлении.

#.Налоксон

371)Какой антагонист следует ввести больному при отравлении морфином?

#.Налоксон

372)В приемное отделение поступил больной без сознания. Пульс - 60 уд. в мин. АД 110/65 мм рт. ст., кожа влажная и холодная, зрачки сужены, рефлексы, кроме сухожильных, отсутствуют, дыхание периодическое. Какой препарат следует ввести пациенту в первую очередь?

#.Налоксон

373)Больному 15 лет, с признаками острого отравления наркотическими анальгетиками: коматозное состояние, зрачки резко сужены, цианоз кожи, в локтевом сгибе свежий след от инъекции, АД 80/60 мм рт.ст., пульс 60 ударов в минуту, дыхание периодическое, рефлексы, кроме сухожильного отсутствуют, требуется назначение специфического антагониста морфина. Назовите его.

#.Налоксон

374)В реанимационное отделение поступил больной с симптомами острого отравления морфином: потеря сознания, гипотермия, дыхание Чейн-Стокса, гипотензия, брадикардия, резкий миоз. Какой из перечисленных препаратов будет наиболее эффективным в данной ситуации?

#.Hалоксон

375)Мужчина 30 лет доставлен в больницу скорой помощи с переломом голени со смещением, выраженным болевым синдромом, который был купирован введением промедола. Каков механизм действия этого препарата?

#.Cтимуляция опиоидных рецепторов ЦНС

376)Больному с неоперабельным раком желудка назначили промедол для снятия выраженного болевого синдрома. Со временем больной отметил уменьшение обезболивающего эффекта и продолжительности действия препарата, резкое усиление болей в организме. Врач объяснил это тем, что:

#.Развилось привыкание

377)Для обезболивания при проведении ортопедического оперативного вмешательства ребенку 4-х лет врач применил промедол. Почему это средство наиболее показано в раннем онтогенезе?

#.Наименее токсичное

378)Ребенок 4-х лет госпитализирован в ортопедическое отделение с переломом голени со смещением. Перед репозицией отломков необходима анальгезия. Какой препарат следует выбрать?

#.Промедол

379)Роженице для обезболивания родов назначен синтетический болеутоляющий препарат, повышающий сократительную активность миометрия и расслабляющий шейку матки. Укажите этот препарат.

#.Промедол

380)Для обезболивания родов врач назначил анальгезирующие средство. Какой из аналгетиков наиболее целесообразно использовать в данной ситуации?

#.Промедол

381)Роженице с патологией беременности необходимо медикаментозное обезболивание родов. Какой препарат можно назначить?

#.Промедол

382)У роженицы 35 лет выражен болевой синдром, связанный с патологией 1-го периода родов. Какое средство наиболее целесообразно применить для обезболивания родов?

#.Промедол

383)В результате падения у ребенка 5 лет возник перелом в нижней трети плечевой кости. Какой препарат наиболее целесообразно использовать для проведения обезболивания в данном случае?

#.Промедол

384)В хирургическое отделение поступил больной 40 лет с диагнозом острый панкреатит. Какой из перечисленных препаратов необходимо назначить с целью уменьшения болевого синдрома и профилактики болевого шока?

#.Промедол

385)У больного с мочекаменной болезнью возникли нестерпимые спастические боли. Для предупреждения болевого шока ему ввели вместе с атропином наркотический анальгетик, который не имеет спазмогенного эффекта. Какой это был препарат?

#.Промедол

386)Укажите наркотический анальгетик синтетического происхождения, превышающий морфин по анальгетической активности в 100-400 раз.

#.Фентанил

387)При инфаркте миокарда была проведена нейролептанальгезия. Какой препарат из группы наркотических анальгетиков чаще всего применяется совместно с дроперидолом?

#.Фентанил

388)Hестероидные противовоспалительные средства назначаются при болях в области головы за исключением:

#.Переломы (травмы) сустава

389)40-летняя женщина обратилась к врачу с жалобой на боль в коленном суставе. При осмотре выявлено: припухлость, покраснение, гипертермия в области сустава. Лабораторное исследование показало позитивные ревмопробы. Какие препараты должны быть назначены для лечения больной?

#.Нестероидные противовоспалительные средства

390)Для патогенетического лечения тригеминальных невропатий, в т.ч. и альвеолярных нервов, используют ненаркотические анальгетики. Определите наиболее достоверный механизм их действия.

#.Ингибирование ЦОГ и угнетение синтеза простагландинов

391)К врачу стоматологу обратилась беременная женщина с диагнозом артрит височно-нижнечелюстного сустава. Какая группа нестероидных противовоспалительных препаратов имеет абсолютные противопоказания к применению у данной больной?

#.Производные салициловой кислоты

392)Аспирин оказывает противовоспалительное действие, так как подавляет активность циклооксигеназы. Уровень каких биологически активных веществ будет при этом снижаться?

#.Простагландинов

393)Больному в подостром периоде инфаркта миокарда врач назначил лекарственный препарат в таблетках, оказывающий антиагрегантное действие, рекомендовав принимать его после еды. Однако больной нарушил предписание врача и стал принимать лекарство бессистемно. Вскоре у него появилась тошнота, рвота, боли в животе. Какой из препаратов мог вызвать эти побочные эффекты?

#.Ацетилсалициловая кислота

394)Ацетилсалициловую кислоту назначают при различных патологических состояниях и с профилактической целью, что основывается на ее фармакологических эффектах. Какой эффект не характерен для указанного средства?

#.Противогистаминный

395)Для лечения хронического ревматоидного артрита больной продолжительное время принимал лечебное средство. Спустя некоторое время развилась общая слабость, недомогание, некротическая ангина. В анализе крови отмечается резкое уменьшение гранулоцитов. Какой препарат мог вызвать такое осложнение?

#.Анальгин

396)Врач-педиатр рекомендовал ребенку при болезненном прорезывании зубов принимать болеутоляющее средство. Мать стала давать лекарственный препарат ребенку при острых респираторных заболеваниях с насморком, лихорадкой, иных недомоганиях. При обследовании ребенка в 3 года, в связи с поступлением в детский сад, анализ крови показал выраженную лейкопению. Какой лекарственный препарат мог вызвать указанное осложнение?

#.Анальгин

397)Больному с невралгией тройничного нерва был введен ненаркотический аналгетик, который имеет быстрое, но кратковременное действие, выпускается в таблетках и ампулах, может вызвать аллергические реакции немедленного типа. Укажите, какой препарат был введен больному.

#.Анальгин

402)К стоматологу обратился пациент с артритом челюстно-лицевого сустава. Врач-стоматолог назначил бутадионовую мазь. Какой механизм действия этой мази?

#.Угнетение циклооксигеназы

399)Больному с артритом височно-челюстного сустава было назначено курсовое лечение таблетками бутадиона. Через две недели лечения больной почувствовал боли в полости рта. При осмотре стоматолог обнаружил язвено-некротический гингивит. С чем связано возникновение этого побочного действия?

#.Угнетение синтеза простагландинов и нарушением синтеза белка клеток слизистой оболочки

400)Больной М., 59 лет, с подагрическим артритом, принимает бутадион. После очередного анализа гемограммы врач отменил этот препарат. Какое осложнение со стороны крови вызвал бутадион?

#.Лейкопению

401)Больной респираторным заболеванием с повышенной температурой длительно применял жаропонижающий препарат. Вскоре появились жалобы на тошноту, боли в эпигастрии и правом подреберье. Какой препарат может вызвать подобные побочные эффекты?

#.Индометацин

402)В медицинской практике широко применяются ненаркотические анальгетики. Они, как правило, обладают болеутоляющим, жаропонижающим и противовоспалительным эффектами, поэтому их часто называют нестероидными противовоспалительными средствами. Укажите ненаркотический анальгетик, который не оказывает противовоспалительное действие.

#.Парацетамол

403)Мужчину 26 лет беспокоят частые головные боли. Известно, что на протяжении 4 лет больной страдает язвенной болезнью. Какой препарат наиболее целесообразно назначить больному для снятия головной боли?

#.Парацетамол

404)Указать препарат обезболивающего типа действия при миозите для больного с язвенной болезнью желудка и лейкопенией:

#.Парацетамол

405)Больному, страдающему мигренью, врач назначил лекарственный препарат. Больной длительно его принимал, после чего стал жаловаться на слабость, тошноту, отеки под глазами по утрам. При обследовании в анализе крови снижено содержание эритроцитов, низкий гемоглобин, в анализе мочи выявлена протеинурия, гематурия, цилиндрурия. Какой из препаратов мог вызвать эти побочные эффекты?

#.Фенацетин

406)Больной ревматоидным полиартритом назначили нестероидный противовоспалительный препарат индометацин. Через некоторое время после его применения у больной возникло обострение сопутствующего заболевания, что вынудило отменить препарат. Какое сопутствующее заболевание могло привести к отмене препарата?

#.Язвенная болезнь

407)При обострении ревматоидного артрита больному, в анамнезе которого сопутствующий хронический гастрит, назначено мелоксикам. Чем бусловлено уменьшение побочного действия препарата на пищеварительный тракт?

#.Преобладающим угнетением циклооксигеназы-2

408)При артрите сустава челюсти стоматолог назначил препарат, который принадлежит к группе ненаркотических анальгетиков и к нестероидным противовоспалительным средствам, преимущественно влияющий на циклооксигеназу-2 и имеющий умеренное раздражающее влияние на слизистую оболочку пищеварительного тракта.

#.Мелоксикам

409)Больной длительно страдает язвенной болезнью 12-перстной кишки. Сегодня в момент подъема тяжести появилась острая интенсивная боль справа в пояснице, усиливающаяся при любом движении. Врач поставил диагноз острый радикулит и назначил с учетом сопутствующего заболевания:

#.Мелоксикам

410)Больному с артритом тазобедренного сустава врач назначил противовоспалительный препарат селективный ингибитор фермента циклооксигеназы-2. Что это за препарат?

#.Мелоксикам

411)В терапевтическое отделение доставлена больная по поводу ревмокардита, в анамнезе которой язвенная болезнь желудка. Какой из перечисленных противовоспалительных препаратов необходимо назначить больной?

#.Мелоксикам

412)При клиническом изучении нового психотропного средства обнаружена способность уменьшать бред и устранять галлюцинации. К какой известной группе психофармакологических средств следует отнести вещество?

#.Антипсихотическим ср-вам

413)Нейролептики имеют способность прекращать бред, галлюцинации, уменьшать агрессивность, ослаблять психомоторное возбуждение. Это действие называется:

#.Антипсихотическое

414)Нейролептиками называют вещества:

#.Устраняющие продуктивные симптомы психоза (бред и галлюцинации)

415)Укажите группу средств, которые не опасны с точки зрения возможности развития лекарственной зависимости.

#.Нейролептиков

416)Укажите группу нейротропных средств, вызывающих при длительном применении лекарственный паркинсонизм.

#.Нейролептики

417)У больного, страдающего шизофренией, на фоне лечения психотропными средствами появилось нарушение координации движений, дрожание рук, сонливость. Какие лекарственные средства наиболее вероятно могли вызвать это состояние?

#.Нейролептики

418)У женщины, длительно принимающей аминазин, возник лептический синдром и значительно выраженный тремор рук. Какой из приведенных механизмов реализован при возникновении этого состояния?

#.Активация гиппокампа

419)Мужчине, страдающему хроническим алкоголизмом, для купирования агрессии и бреда была сделана внутривенно инъекция аминазина, что вызвало у больного потерю сознания. Какова наиболее вероятная причина возникшего осложнения?

#.Ортостатический коллапс

420)Больному с псхомоторным возбуждением после внутримышечного введения 2 мл 2,5% раствора аминазина наступило резкое снижение АД (преколлаптоидное состояние). Что необходимо ввести больному для нормализации АД?

#.Кордиамин

421)Какой эффект не возникает при введении аминазина?

#.Стимуляция коры большого мозга

422)Для снятия бреда и галлюцинаций у больной шизофренией врач использовал аминазин. Какой механизм антипсихотического действия препарата?

#.Блокада адренергических и дофаминергических процессов в ЦНС

423)Больному шизофренией был назначен аминазин. Какой из перечисленных фармакодинамических эффектов является основанием для его назначения данному больному?

#.Антипсихотический

424)Укажите средство для купирования острого психоза из группы производных фенотиазина.

#.Аминазин

425)В психиатрическую клинику доставлен мужчина 40 лет в состоянии возбуждения. Агрессивен, бредит. Какое лекарство наиболее целесообразно ввести больному?

#.Аминазин

426)Больному с явлениями острого психоза, сопровождающегося бредом и галлюцинациями, врач ввел в/в лекарственный препарт. Вскоре явления психоза были устранены, но когда больной встал с постели, он внезапно побледнел и потерял сознание. Был поставлен диагноз ортостатического коллапса. Какой из перечисленных препаратов наиболее часто вызывает указанное осложнение?

#.Аминазин

427)Больной на протяжении двух недель получал нейролептическое средство. Состояние больного улучшилось, однако вскоре появилась ригидность, тремор, гипокинезия. Какой из перечисленных препаратов вызывает указанные осложнения?

#.Аминазин

428)На приеме у стоматолога у больной 65 лет на фоне выраженного страха и беспокойства внезапно появилась сдавливающая боль в затылке, головокружение, мелькание "мушек" перед глазами, АД 190/100 мм. рт.ст. Какой из нижеперечисленных препаратов необходимо применить для оказания неотложной помощи?

#.Аминазин

429)У медицинской сестры после двухлетней работы в психиатрическом отделении на руках развился нейродерматит. Какой препарат вызывает это побочное действие у медперсонала?

#.Аминазин

430)При работе с аминазином у медперсонала возможное следующее осложнение:

#.Контактный дерматит

431)Для проведения оперативного вмешательства необходимо использовать метод общего охлаждения. При использовании какого препарата в комбинации с физическим охлаждением наблюдается значительная гипотермия?

#.Аминазин

432)Животным разных видов вводили эквивалентные дозы аминазина. У какого из названных видов следует ожидать наибольшего снижения температуры тела?

#.Мышь

433)В психиатрической клинике используют трифтазин для купирования психоза. Какой из перечисленных механизмов антипсихотического действия принадлежит этому средству?

#.Угнетение дофаминовых D2-рецепторов

434)Больному для лечения психотического состояния был назначен трифтазин. Какой механизм его антипсихотического действия?

#.Угнетение дофаминовых D2-рецепторов

435)После длительного применения аминазина в связи с развитием побочных эффектов врач назначил пациенту другой препарат из группы нейролептиков. Какой из перечисленных препаратов относится к этой группе средств?

#.Галоперидол

436)Больному с алкогольным психозом назначен нейролептик И производное бутирофенона. Какой из перечисленных препаратов относится к этой группе средств?

#.Галоперидол

437)У больного имеются явления психоза в виде психомоторного возбуждения, слуховых и зрительных галлюцинаций. Отметить препарат, показанный при описанном состоянии.

#.Галоперидол

438)При инфаркте миокарда была проведена нейролептанальгезия. Какой препарат из группы нейролептиков чаще всего применяется совместно с фентанилом?

#.Дроперидол

439)У больного диагностирова острый инфаркт миокарда, сопровождающийся стойкими болями за грудиной. Неэффективность ранеее введенных препаратов дала основание врачу провести нейролептанальгезию. Какой нейролептик надо использовать?

#.Дроперидол

440)У больного острая гипотензия вызванная передозировкой дроперидола. Какой из предлагаемых веществ необходим для купирования этого состояния?

#.Ангиотензинамид

441)У больных, принимающих длительное время нейролептики, частым осложнением является паркинсонизм. При появлении первых признаков этого заболевании препарат отменяют. Какой препарат из группы нейролептиков длительное время не вызывает явлений паркинсонизма и его можно использовать для замены лечебного средства, что вызывал паркинсонизм?

#.Клозапин

442)Больному со слабо выраженными проявлениями синдрома Паркинсона предстоит длительное применение нейролептика, большинство которых вызывает и усиливает названную патологию. Какой нейролептик наименее опасен в таком случае?

#.Клозапин

443)Укажите, за счет чего соли лития предупреждают у больных маниакальное возбуждение.

#.Уменьшают содержание норадреналина в синапсах ЦНС за счет повышения внутриклеточного окислительного дезаминирования и более активного нейронального захвата медиатора

444)При экспериментальном изучении нового вещества у него обнаружена способность понижать тонус скелетных мышц и подавлять чувство страха и тревоги. К какой группе фармакологических средств его можно отнести?

#.Транквилизаторам

445)Больной обратился к врачу с жалобами на эмоциональную неустойчивость, чувство психо-эмоционального напряжения и страха при встрече со своим начальником, сопровождающееся сильным сердцебиением и болями в области сердца, покраснением лица, головной болью, тремором рук, потливостью. Врач поставил диагноз невроза с выраженным вегето-сосудистым компонентом. Назначьте группу лекарственных препаратов, учитывая состояние больного.

#.Транквилизаторы

446)При проведении плановой операции в области верхней челюсти хирург решил использовать атаральгезию. Какие лекарственные средства используют для этой манипуляции?

#.Транквилизаторы

447)Больному для проведения оперативного вмешательства была проведена атаралгезия. Комбинация каких препаратов используентся для этого вида анестезии?

#.Диазепам + промедол

448)Женщина 35 лет обратилась к врачу с жалобами на раздражительность, быструю усталость, повышенную чувствительность, бессонницу. Для устранения невроза врач назначил пациентке транквилизатор диазепам. Укажите фармакодинамический эффект диазепама, который дал возможность применить его в данных обстоятельствах.

#.Анксиолитический

449)Назовите группу препаратов, длительный прием которых приводит к депрессии, увеличению аварий на транспорте, снижению трудоспособности, нарушению структуры сна.

#.Бензодиазепины

450)Мужчине с нарушениями психоэмоциональной сферы и расстройством сна был назначен диазепам. С влиянием на какие рецепторы связано действие препарата?

#.Бензодиазепиновые рецепторы

451)Женщине 48 лет перед экстракцией зуба ввели раствор сибазона. Чем объясняется анксиолитическое действие данного препарата?

#.Стимуляция бензодиазепиновых рецепторов

456)При челюстно-лицевой операции в стационар, врач-стоматолог для премедикации с целью уменьшения чувства страха и для патенцирования действия анестетиков, назначил больному транквилизатор производное бензодиазепина. Какой из перечисленных препаратов назначил врач?

#.Диазепам

452)Больной обратился к врачу с жалобами на тревогу, страх, ощущение беспокойства, внутреннее напряжение. Какой препарат следует ему назначить?

#.Диазепам

453)Студент обратился к врачу с просьбой помочь ему перебороть страх перед стоматологическими манипуляциями. Врач предложил ему принять препарат:

#.Диазепам

454)Стоматологическому больному для снятия чувства страха перед болью назначили седативный препарат. Какой препарат наиболее эффективен в данном случае?

#.Седуксен

455)Во время ожидания экстрации зуба у пациента возникло сильное чувство тревоги и страха. Какой из лекарственных препаратов ему необходимо принять для устранения этого ощущения?

#.Хлозепид

456)Больной с повышенной раздражительностью, плаксивостью, бессонницей назначили феназепам. Укажите его механизм действия.

#.Стимуляция бензодиазепинових рецепторов

457)У больного отмечается повышенная возбудимость, бессонница, беспричинная тревога, появился страх высоты. Какое лекарственное средство целесообразно назначить?

#.Феназепам

458)У водителя грузовика, мужчины 50 лет, выявлена неврастения. Укажите "дневной" транквилизатор, назначаемый при амбулаторном лечении.

#.Триоксазин

459)У больного после длительного применения бензодиазепинов появились жалобы на сонливость, замедленные двигательные реакции, нарушения памяти, слабость, головная боль, тошнота, кожные высыпания. Врач для ликвидации этих явлений назначил препарат с фармакодинамическим типом взаимодействия (антагонист). Определите препарат, назначенный врачом.

#.Флумазенил

460)После обследования пациента врач рекомендовал ему средство из группы препаратов, использующихся при неврозах и неврозоподобных состояниях, повышенной возбудимости и обидчивости. Препарат какой группы назначен больному?

#.Седативные

461)Больной обратился к врачу с жалобами на раздражительность, бессонницу, быструю утомляемость. В качестве успокаивающего средства ему был назначен лекарственный препарат. Вскоре указанные симптомы исчезли, но больной продолжал принимать лекарство с профилактической целью. Через некоторое время у него появились насморк, кашель, конъюктивит, дерматит, больной стал отмечать сонливость, ослабление памяти. Врач поставил диагноз кумуляции данного лекарственного препарата и назначил лечение хлоридом натрия и обильное питье. Какой препарат принимал больной?

#.Натрия бромид

462)Больная неврастенией в течение 2 недель принимала назначенную невропатологом микстуру. Самочувствие несколько улучшилось, однако пояились жалобы на насморк, конъюнктивит, кожные высыпания, вялость и ослабление памяти. Препараты какой группы могут вызывать описанные побочные явления?

#.Соли брома

463)Больному с жалобами на повышенную раздражительность, бессонницу и быструю утомляемость был назначен препарат из группы седативных средств. Через несколько дней после применения препарата у больного появились насморк, кашель, коньюнктивит, дерматит, сонливость, Какой препарат назначил врач?

#.Hатрия бромид

464)К врачу обратился больной с жалобами на недомогание, угнетенное настроение, появление прыщей на коже. При опросе выяснилось, что больной на протяжении последних 3 недель принимал микстуру, выписанную невропатологом. С приемом препаратов какой группы связаны жалобы пациента?

#.Бромиды

465)Больная неврастенией в течение 2 недель принимала назначенную невропатологом микстуру. Самочувствие несколько улучшилось, однако появились жалобы на насморк, конъюнктивит, кожные высыпания, вялость и ослабление памяти. Был поставлен диагноз: бромизм. Что целесообразно назначить для скорейшего устранения указанных симптомов?

#.Натрия хлорид

466)Мужчину 40 лет на протяжении месяца беспокоит насморк, кашель, слезотечение. Температура тела нормальная. Апатичен. При осмотре на коже туловища имеется угревая сыпь. Выяснено, что в течение 3 месяцев больной принимает бромиды. Какое средство целесообразно назначить больному для ускорения выведения бромидов из организма?

#.Поваренную соль

467)Больному с признаками переутомления, выражающегося в понижении настроения и психомоторной активности, снижении умственной и физической работоспособности с сопутствующей артериальной гипертензией необходимо назначить препатат из групы психостимуляторов. Какой препарат из перечисленных нельзя назначить больному с учетом сопутствующего заболевания.

#.Фенамин

468)У больной Ю. 59 лет с нейроциркуляторной (первичной) артериальной гипотензией купирован гипотонический криз подкожным введением 1 мл 20% раствора кофеина бензоата натрия. Механизм действия этого препарата?

#.Блокада аденозиновых рецепторов

469)Укажите показание к применению кофеина.

#.Повышенная сонливость

470)Для фармакодинамики какого лекарственного средства характерно повышение настроения и психомоторной активности, снижение чувства утомления, увеличесние физической и умственной работоспособности, временное снижение потребности во сне?

#.Кофеин

471)Укажите средство, сочетающее в себе свойства аналептика и психостимулятора.

#.Кофеин

472)Машиной скорой помощи в больницу доставили больного, который в состоянии тяжелой депрессии старался покончить жизнь самоубийством. Диагноз: депрессивный психоз. Какие фармакологические средства необходимо назначить больному?

#.Антидепрессанты

Соседние файлы в предмете [НЕСОРТИРОВАННОЕ]